What do high BUN/Cr ratios mean?

Article Type
Changed
Thu, 03/09/2023 - 11:07

A 43-year-old man presents to the emergency department with dizziness and a 6-hour history of passing maroon stool. He has been in good health with the only medical problem in his history being depression.

He is taking sertraline. On exam, his blood pressure is 100/60, and his pulse is 100, both while lying down. His blood pressure while standing is 90/60 and his pulse while standing is 130. The rest of his exam is normal. His lab values include hemoglobin of 10, hematocrit of 30, white blood cell of 4.6, platelet count of 175,000, sodium of 142, chloride of 100, bicarbonate of 24, potassium of 3.8, blood urea nitrogen (BUN) of 38, and creatinine clearance (Cr) of 1.1.

Dr. Paauw

What is the most likely source of his bleeding?

A. Gastric ulcer

B. Meckel’s diverticulum

C. Arteriovenous malformation

D. Diverticulosis

E. Hemorrhoids

What makes the most sense

The most likely cause of this patient’s maroon stool is an upper gastrointestinal bleed, so it would make the most sense for a gastric ulcer to be the source of his bleeding. The clue here is the very high BUN/Cr ratio.

We were all taught early in our training that a high BUN/Cr ratio represented volume depletion. This is certainly the most common cause, but very high BUN/Cr ratios (over 30) can represent causes beyond volume depletion.

Witting and colleagues studied factors that predicted upper GI bleeding in patients presenting without hematemesis. They found that the three strongest predictors were black stool (odds ratio, 16.6), BUN/Cr ratio greater than 30 (OR, 10), and age greater than 50 (OR, 8.4).1

Srygley and colleagues reviewed high-quality studies of factors associated with upper GI bleeding.2 Factors that were found to increase the likelihood of an upper gastrointestinal bleed were Melenic stool on exam (likelihood ratio, 25), blood or coffee grounds on nasogastric aspiration (LR, 9.6), and BUN/Cr ratio greater than 30 (LR, 7.5).

Very high BUN/Cr ratios can indicate problems other than UGI bleeding and volume depletion. High BUN/Cr ratios are seen in patients with heart failure.

Zhang and colleagues studied if a high BUN/Cr ratio helped distinguish heart failure from asthma and chronic obstructive pulmonary disease (COPD).3 They found that, compared with those in the asthma group, the BUN/Cr ratios were significantly increased in the heart failure group (P < .05), whereas no significant differences in BUN/Cr ratios were found between the asthma and COPD groups.

Cheang and colleagues conducted their own study, as well as a meta-analysis, looking to see if high BUN/Cr ratios predicted increased mortality in patients with acute heart failure.4 In the meta-analysis of 8 studies (including their own), they found that the highest BUN/Cr ratio category was associated with an 77% higher all-cause mortality than the lowest category (hazard ratio, 1.77; 95% confidence interval, 1.52-2.07).

High dose corticosteroids can raise BUN levels, especially in patients with chronic kidney disease, and cause unexpectedly high BUN/Cr ratios.
 

Pearl

Very high BUN/Cr ratios (greater than 30) can signify upper GI bleeding, heart failure, or high-dose corticosteroid use.

Dr. Paauw is professor of medicine in the division of general internal medicine at the University of Washington, Seattle, and he serves as third-year medical student clerkship director at the University of Washington. Contact Dr. Paauw at [email protected].

References

1. Am J Emerg Med. 2006 May;24(3):280-5.

2. JAMA. 2012;307(10):1072-9.

3. Comput Math Methods Med. 2022 Jul 21. doi: 10.1155/2022/4586458.

4. Cardiorenal Med. 2020;10:415-28.

Publications
Topics
Sections

A 43-year-old man presents to the emergency department with dizziness and a 6-hour history of passing maroon stool. He has been in good health with the only medical problem in his history being depression.

He is taking sertraline. On exam, his blood pressure is 100/60, and his pulse is 100, both while lying down. His blood pressure while standing is 90/60 and his pulse while standing is 130. The rest of his exam is normal. His lab values include hemoglobin of 10, hematocrit of 30, white blood cell of 4.6, platelet count of 175,000, sodium of 142, chloride of 100, bicarbonate of 24, potassium of 3.8, blood urea nitrogen (BUN) of 38, and creatinine clearance (Cr) of 1.1.

Dr. Paauw

What is the most likely source of his bleeding?

A. Gastric ulcer

B. Meckel’s diverticulum

C. Arteriovenous malformation

D. Diverticulosis

E. Hemorrhoids

What makes the most sense

The most likely cause of this patient’s maroon stool is an upper gastrointestinal bleed, so it would make the most sense for a gastric ulcer to be the source of his bleeding. The clue here is the very high BUN/Cr ratio.

We were all taught early in our training that a high BUN/Cr ratio represented volume depletion. This is certainly the most common cause, but very high BUN/Cr ratios (over 30) can represent causes beyond volume depletion.

Witting and colleagues studied factors that predicted upper GI bleeding in patients presenting without hematemesis. They found that the three strongest predictors were black stool (odds ratio, 16.6), BUN/Cr ratio greater than 30 (OR, 10), and age greater than 50 (OR, 8.4).1

Srygley and colleagues reviewed high-quality studies of factors associated with upper GI bleeding.2 Factors that were found to increase the likelihood of an upper gastrointestinal bleed were Melenic stool on exam (likelihood ratio, 25), blood or coffee grounds on nasogastric aspiration (LR, 9.6), and BUN/Cr ratio greater than 30 (LR, 7.5).

Very high BUN/Cr ratios can indicate problems other than UGI bleeding and volume depletion. High BUN/Cr ratios are seen in patients with heart failure.

Zhang and colleagues studied if a high BUN/Cr ratio helped distinguish heart failure from asthma and chronic obstructive pulmonary disease (COPD).3 They found that, compared with those in the asthma group, the BUN/Cr ratios were significantly increased in the heart failure group (P < .05), whereas no significant differences in BUN/Cr ratios were found between the asthma and COPD groups.

Cheang and colleagues conducted their own study, as well as a meta-analysis, looking to see if high BUN/Cr ratios predicted increased mortality in patients with acute heart failure.4 In the meta-analysis of 8 studies (including their own), they found that the highest BUN/Cr ratio category was associated with an 77% higher all-cause mortality than the lowest category (hazard ratio, 1.77; 95% confidence interval, 1.52-2.07).

High dose corticosteroids can raise BUN levels, especially in patients with chronic kidney disease, and cause unexpectedly high BUN/Cr ratios.
 

Pearl

Very high BUN/Cr ratios (greater than 30) can signify upper GI bleeding, heart failure, or high-dose corticosteroid use.

Dr. Paauw is professor of medicine in the division of general internal medicine at the University of Washington, Seattle, and he serves as third-year medical student clerkship director at the University of Washington. Contact Dr. Paauw at [email protected].

References

1. Am J Emerg Med. 2006 May;24(3):280-5.

2. JAMA. 2012;307(10):1072-9.

3. Comput Math Methods Med. 2022 Jul 21. doi: 10.1155/2022/4586458.

4. Cardiorenal Med. 2020;10:415-28.

A 43-year-old man presents to the emergency department with dizziness and a 6-hour history of passing maroon stool. He has been in good health with the only medical problem in his history being depression.

He is taking sertraline. On exam, his blood pressure is 100/60, and his pulse is 100, both while lying down. His blood pressure while standing is 90/60 and his pulse while standing is 130. The rest of his exam is normal. His lab values include hemoglobin of 10, hematocrit of 30, white blood cell of 4.6, platelet count of 175,000, sodium of 142, chloride of 100, bicarbonate of 24, potassium of 3.8, blood urea nitrogen (BUN) of 38, and creatinine clearance (Cr) of 1.1.

Dr. Paauw

What is the most likely source of his bleeding?

A. Gastric ulcer

B. Meckel’s diverticulum

C. Arteriovenous malformation

D. Diverticulosis

E. Hemorrhoids

What makes the most sense

The most likely cause of this patient’s maroon stool is an upper gastrointestinal bleed, so it would make the most sense for a gastric ulcer to be the source of his bleeding. The clue here is the very high BUN/Cr ratio.

We were all taught early in our training that a high BUN/Cr ratio represented volume depletion. This is certainly the most common cause, but very high BUN/Cr ratios (over 30) can represent causes beyond volume depletion.

Witting and colleagues studied factors that predicted upper GI bleeding in patients presenting without hematemesis. They found that the three strongest predictors were black stool (odds ratio, 16.6), BUN/Cr ratio greater than 30 (OR, 10), and age greater than 50 (OR, 8.4).1

Srygley and colleagues reviewed high-quality studies of factors associated with upper GI bleeding.2 Factors that were found to increase the likelihood of an upper gastrointestinal bleed were Melenic stool on exam (likelihood ratio, 25), blood or coffee grounds on nasogastric aspiration (LR, 9.6), and BUN/Cr ratio greater than 30 (LR, 7.5).

Very high BUN/Cr ratios can indicate problems other than UGI bleeding and volume depletion. High BUN/Cr ratios are seen in patients with heart failure.

Zhang and colleagues studied if a high BUN/Cr ratio helped distinguish heart failure from asthma and chronic obstructive pulmonary disease (COPD).3 They found that, compared with those in the asthma group, the BUN/Cr ratios were significantly increased in the heart failure group (P < .05), whereas no significant differences in BUN/Cr ratios were found between the asthma and COPD groups.

Cheang and colleagues conducted their own study, as well as a meta-analysis, looking to see if high BUN/Cr ratios predicted increased mortality in patients with acute heart failure.4 In the meta-analysis of 8 studies (including their own), they found that the highest BUN/Cr ratio category was associated with an 77% higher all-cause mortality than the lowest category (hazard ratio, 1.77; 95% confidence interval, 1.52-2.07).

High dose corticosteroids can raise BUN levels, especially in patients with chronic kidney disease, and cause unexpectedly high BUN/Cr ratios.
 

Pearl

Very high BUN/Cr ratios (greater than 30) can signify upper GI bleeding, heart failure, or high-dose corticosteroid use.

Dr. Paauw is professor of medicine in the division of general internal medicine at the University of Washington, Seattle, and he serves as third-year medical student clerkship director at the University of Washington. Contact Dr. Paauw at [email protected].

References

1. Am J Emerg Med. 2006 May;24(3):280-5.

2. JAMA. 2012;307(10):1072-9.

3. Comput Math Methods Med. 2022 Jul 21. doi: 10.1155/2022/4586458.

4. Cardiorenal Med. 2020;10:415-28.

Publications
Publications
Topics
Article Type
Sections
Disallow All Ads
Content Gating
No Gating (article Unlocked/Free)
Alternative CME
Disqus Comments
Default
Use ProPublica
Hide sidebar & use full width
render the right sidebar.
Conference Recap Checkbox
Not Conference Recap
Clinical Edge
Display the Slideshow in this Article
Medscape Article
Display survey writer
Reuters content
Disable Inline Native ads
WebMD Article

What impact do carbs have on bone health?

Article Type
Changed
Tue, 03/14/2023 - 17:46

I am often asked about the impact of dietary nutrients on bone health, particularly as many patients with low bone density, many with a history of multiple fractures, are referred to me. Many factors affect bone density, an important predictor of fracture risk, including genetics, body weight and muscle mass, bone loading exercise, menstrual status, other hormonal factors, nutritional status, optimal absorption of dietary nutrients, and medication use.

Dietary nutrients include macronutrients (carbohydrates, proteins, fat, and fiber) and micronutrients (such as dietary minerals and vitamins). The importance of micronutrients such as calcium, phosphorus, magnesium, and vitamins C, D, and K in optimizing bone mineralization and bone formation has been well documented.

The impact of protein intake on bone health is slightly more controversial, with some studies suggesting that increased protein intake may be deleterious to bone by increasing acid load, which in turn, increases calcium loss in urine. Overall data analysis from multiple studies support the finding that a higher protein intake is modestly beneficial for bone at certain sites, such as the spine.

Though data regarding the impact of dietary carbohydrates on bone are not as robust, it’s important to understand these effects given the increasing knowledge of the deleterious impact of processed carbohydrates on weight and cardiometabolic outcomes. This leads to the growing recommendations to limit carbohydrates in diet.
 

Quality and quantity of carbs affect bone health

Available studies suggest that both the quality and quantity of carbohydrates that are in a diet as well as the glycemic index of food may affect bone outcomes. Glycemic index refers to the extent of blood glucose elevation that occurs after the intake of any specific food. Foods with a higher glycemic index cause a rapid increase in blood glucose, whereas those with a low glycemic index result in a slower and more gradual increase. Examples of high–glycemic index food include processed and baked foods (such as breakfast cereals [unless whole grain], pretzels, cookies, doughnuts, pastries, cake, white bread, bagels, croissants, and corn chips), sugar-sweetened beverages, white rice, fast food (such as pizza and burgers), and potatoes. Examples of low glycemic index foods include vegetables, fruits, legumes, dairy and dairy products (without added sugar), whole-grain foods (such as oat porridge), and nuts.

A high–glycemic index diet has been associated with a greater risk for obesity and cardiovascular disease, and with lower bone density, an increased risk for fracture. This has been attributed to acute increases in glucose and insulin levels after consumption of high–glycemic index food, which causes increased oxidative stress and secretion of inflammatory cytokines, such as interleukin 6 and tumor necrosis factor alpha, that activate cells in bone that increase bone loss.

Higher blood glucose concentrations induced by a higher dietary glycemic index can have deleterious effects on osteoblasts, the cells important for bone formation, and increase bone loss through production of advanced glycation end products that affect the cross linking of collagen in bone (important for bone strength), as well as calcium loss in urine. This was recently reported in a study by Garcia-Gavilan and others, in which the authors showed that high dietary glycemic index and dietary glucose load are associated with a higher risk for osteoporosis-related fractures in an older Mediterranean population who are at high risk for cardiovascular events. Similar data were reported by Nouri and coauthors in a study from Iran.

The quantity and quality of dietary carbohydrates may also have an impact on bone. The quality of carbohydrates has been assessed using the carbohydrate quality index (CQI) and the low carbohydrate diet score (LCDS). The CQI takes into account dietary fiber intake, glycemic index, intake of processed vs. whole grain, and solid vs. total carbohydrates in diet. A higher CQI diet is associated with reduced cardiovascular risk. Higher LCDS reflects lower carbohydrate and higher fat and protein intake.

Diets that are rich in refined or processed carbohydrates with added sugar are proinflammatory and increase oxidative stress, which may lead to increased bone loss, low bone density, and increased fracture risk. These foods also have a high glycemic index.

In contrast, diets that are rich in whole grains, legumes, fruits, vegetables, nuts, and olive oil have a lower glycemic index and are beneficial to bone. These diets have a higher CQI and LCDS (as reported by Nouri and coauthors) and provide a rich source of antioxidants, vitamins, minerals, and other nutrients (such as calcium, magnesium, and vitamins B, C, and K), which are all beneficial to bone. Gao and others have reported that implementing a low glycemic index pulse-based diet (lentils, peas, beans) is superior to a regular hospital diet in preventing the increase in bone loss that typically occurs during hospitalization with enforced bed rest.

Most reports of the impact of carbohydrates on bone health are from observational studies. In an interventional study, Dalskov and coauthors randomly assigned children aged 5-18 years who had parents with overweight to one of five diets (high protein/low glycemic index, high protein/high glycemic index, low protein/low glycemic index, low protein/high glycemic index, or regular) for 6 months.

Contrasting with our understanding that protein intake is overall good for bone, this study found that among patients receiving a high–glycemic index diet, those who were on a high-protein diet had greater reductions in a bone formation marker than did those on a low-protein diet, with no major changes observed with the other diets. This suggests the influence of associated dietary nutrients on bone outcomes and that protein intake may modify the effects of dietary carbohydrates on bone formation. Similarly, the fat content of food can alter the glycemic index and thus may modify the impact of dietary carbohydrates on bone.

In summary, available data suggest that the quantity and quality of carbohydrates, including the glycemic index of food, may affect bone health and that it is important to exercise moderation in the consumption of such foods. However, there are only a few studies that have examined these associations, and more studies are necessary to further clarify the impact of dietary carbohydrates on bone as well as any modifications of these effects by other associated food groups. These studies will allow us to refine our recommendations to our patients as we advance our understanding of the impact of the combined effects of various dietary nutrients on bone.

Madhusmita Misra, MD, MPH, is chief of the division of pediatric endocrinology, Mass General for Children, Boston, and serves or has served as a director, officer, partner, employee, advisor, consultant, or trustee for AbbVie, Sanofi, and Ipsen.

A version of this article first appeared on Medscape.com.

Publications
Topics
Sections

I am often asked about the impact of dietary nutrients on bone health, particularly as many patients with low bone density, many with a history of multiple fractures, are referred to me. Many factors affect bone density, an important predictor of fracture risk, including genetics, body weight and muscle mass, bone loading exercise, menstrual status, other hormonal factors, nutritional status, optimal absorption of dietary nutrients, and medication use.

Dietary nutrients include macronutrients (carbohydrates, proteins, fat, and fiber) and micronutrients (such as dietary minerals and vitamins). The importance of micronutrients such as calcium, phosphorus, magnesium, and vitamins C, D, and K in optimizing bone mineralization and bone formation has been well documented.

The impact of protein intake on bone health is slightly more controversial, with some studies suggesting that increased protein intake may be deleterious to bone by increasing acid load, which in turn, increases calcium loss in urine. Overall data analysis from multiple studies support the finding that a higher protein intake is modestly beneficial for bone at certain sites, such as the spine.

Though data regarding the impact of dietary carbohydrates on bone are not as robust, it’s important to understand these effects given the increasing knowledge of the deleterious impact of processed carbohydrates on weight and cardiometabolic outcomes. This leads to the growing recommendations to limit carbohydrates in diet.
 

Quality and quantity of carbs affect bone health

Available studies suggest that both the quality and quantity of carbohydrates that are in a diet as well as the glycemic index of food may affect bone outcomes. Glycemic index refers to the extent of blood glucose elevation that occurs after the intake of any specific food. Foods with a higher glycemic index cause a rapid increase in blood glucose, whereas those with a low glycemic index result in a slower and more gradual increase. Examples of high–glycemic index food include processed and baked foods (such as breakfast cereals [unless whole grain], pretzels, cookies, doughnuts, pastries, cake, white bread, bagels, croissants, and corn chips), sugar-sweetened beverages, white rice, fast food (such as pizza and burgers), and potatoes. Examples of low glycemic index foods include vegetables, fruits, legumes, dairy and dairy products (without added sugar), whole-grain foods (such as oat porridge), and nuts.

A high–glycemic index diet has been associated with a greater risk for obesity and cardiovascular disease, and with lower bone density, an increased risk for fracture. This has been attributed to acute increases in glucose and insulin levels after consumption of high–glycemic index food, which causes increased oxidative stress and secretion of inflammatory cytokines, such as interleukin 6 and tumor necrosis factor alpha, that activate cells in bone that increase bone loss.

Higher blood glucose concentrations induced by a higher dietary glycemic index can have deleterious effects on osteoblasts, the cells important for bone formation, and increase bone loss through production of advanced glycation end products that affect the cross linking of collagen in bone (important for bone strength), as well as calcium loss in urine. This was recently reported in a study by Garcia-Gavilan and others, in which the authors showed that high dietary glycemic index and dietary glucose load are associated with a higher risk for osteoporosis-related fractures in an older Mediterranean population who are at high risk for cardiovascular events. Similar data were reported by Nouri and coauthors in a study from Iran.

The quantity and quality of dietary carbohydrates may also have an impact on bone. The quality of carbohydrates has been assessed using the carbohydrate quality index (CQI) and the low carbohydrate diet score (LCDS). The CQI takes into account dietary fiber intake, glycemic index, intake of processed vs. whole grain, and solid vs. total carbohydrates in diet. A higher CQI diet is associated with reduced cardiovascular risk. Higher LCDS reflects lower carbohydrate and higher fat and protein intake.

Diets that are rich in refined or processed carbohydrates with added sugar are proinflammatory and increase oxidative stress, which may lead to increased bone loss, low bone density, and increased fracture risk. These foods also have a high glycemic index.

In contrast, diets that are rich in whole grains, legumes, fruits, vegetables, nuts, and olive oil have a lower glycemic index and are beneficial to bone. These diets have a higher CQI and LCDS (as reported by Nouri and coauthors) and provide a rich source of antioxidants, vitamins, minerals, and other nutrients (such as calcium, magnesium, and vitamins B, C, and K), which are all beneficial to bone. Gao and others have reported that implementing a low glycemic index pulse-based diet (lentils, peas, beans) is superior to a regular hospital diet in preventing the increase in bone loss that typically occurs during hospitalization with enforced bed rest.

Most reports of the impact of carbohydrates on bone health are from observational studies. In an interventional study, Dalskov and coauthors randomly assigned children aged 5-18 years who had parents with overweight to one of five diets (high protein/low glycemic index, high protein/high glycemic index, low protein/low glycemic index, low protein/high glycemic index, or regular) for 6 months.

Contrasting with our understanding that protein intake is overall good for bone, this study found that among patients receiving a high–glycemic index diet, those who were on a high-protein diet had greater reductions in a bone formation marker than did those on a low-protein diet, with no major changes observed with the other diets. This suggests the influence of associated dietary nutrients on bone outcomes and that protein intake may modify the effects of dietary carbohydrates on bone formation. Similarly, the fat content of food can alter the glycemic index and thus may modify the impact of dietary carbohydrates on bone.

In summary, available data suggest that the quantity and quality of carbohydrates, including the glycemic index of food, may affect bone health and that it is important to exercise moderation in the consumption of such foods. However, there are only a few studies that have examined these associations, and more studies are necessary to further clarify the impact of dietary carbohydrates on bone as well as any modifications of these effects by other associated food groups. These studies will allow us to refine our recommendations to our patients as we advance our understanding of the impact of the combined effects of various dietary nutrients on bone.

Madhusmita Misra, MD, MPH, is chief of the division of pediatric endocrinology, Mass General for Children, Boston, and serves or has served as a director, officer, partner, employee, advisor, consultant, or trustee for AbbVie, Sanofi, and Ipsen.

A version of this article first appeared on Medscape.com.

I am often asked about the impact of dietary nutrients on bone health, particularly as many patients with low bone density, many with a history of multiple fractures, are referred to me. Many factors affect bone density, an important predictor of fracture risk, including genetics, body weight and muscle mass, bone loading exercise, menstrual status, other hormonal factors, nutritional status, optimal absorption of dietary nutrients, and medication use.

Dietary nutrients include macronutrients (carbohydrates, proteins, fat, and fiber) and micronutrients (such as dietary minerals and vitamins). The importance of micronutrients such as calcium, phosphorus, magnesium, and vitamins C, D, and K in optimizing bone mineralization and bone formation has been well documented.

The impact of protein intake on bone health is slightly more controversial, with some studies suggesting that increased protein intake may be deleterious to bone by increasing acid load, which in turn, increases calcium loss in urine. Overall data analysis from multiple studies support the finding that a higher protein intake is modestly beneficial for bone at certain sites, such as the spine.

Though data regarding the impact of dietary carbohydrates on bone are not as robust, it’s important to understand these effects given the increasing knowledge of the deleterious impact of processed carbohydrates on weight and cardiometabolic outcomes. This leads to the growing recommendations to limit carbohydrates in diet.
 

Quality and quantity of carbs affect bone health

Available studies suggest that both the quality and quantity of carbohydrates that are in a diet as well as the glycemic index of food may affect bone outcomes. Glycemic index refers to the extent of blood glucose elevation that occurs after the intake of any specific food. Foods with a higher glycemic index cause a rapid increase in blood glucose, whereas those with a low glycemic index result in a slower and more gradual increase. Examples of high–glycemic index food include processed and baked foods (such as breakfast cereals [unless whole grain], pretzels, cookies, doughnuts, pastries, cake, white bread, bagels, croissants, and corn chips), sugar-sweetened beverages, white rice, fast food (such as pizza and burgers), and potatoes. Examples of low glycemic index foods include vegetables, fruits, legumes, dairy and dairy products (without added sugar), whole-grain foods (such as oat porridge), and nuts.

A high–glycemic index diet has been associated with a greater risk for obesity and cardiovascular disease, and with lower bone density, an increased risk for fracture. This has been attributed to acute increases in glucose and insulin levels after consumption of high–glycemic index food, which causes increased oxidative stress and secretion of inflammatory cytokines, such as interleukin 6 and tumor necrosis factor alpha, that activate cells in bone that increase bone loss.

Higher blood glucose concentrations induced by a higher dietary glycemic index can have deleterious effects on osteoblasts, the cells important for bone formation, and increase bone loss through production of advanced glycation end products that affect the cross linking of collagen in bone (important for bone strength), as well as calcium loss in urine. This was recently reported in a study by Garcia-Gavilan and others, in which the authors showed that high dietary glycemic index and dietary glucose load are associated with a higher risk for osteoporosis-related fractures in an older Mediterranean population who are at high risk for cardiovascular events. Similar data were reported by Nouri and coauthors in a study from Iran.

The quantity and quality of dietary carbohydrates may also have an impact on bone. The quality of carbohydrates has been assessed using the carbohydrate quality index (CQI) and the low carbohydrate diet score (LCDS). The CQI takes into account dietary fiber intake, glycemic index, intake of processed vs. whole grain, and solid vs. total carbohydrates in diet. A higher CQI diet is associated with reduced cardiovascular risk. Higher LCDS reflects lower carbohydrate and higher fat and protein intake.

Diets that are rich in refined or processed carbohydrates with added sugar are proinflammatory and increase oxidative stress, which may lead to increased bone loss, low bone density, and increased fracture risk. These foods also have a high glycemic index.

In contrast, diets that are rich in whole grains, legumes, fruits, vegetables, nuts, and olive oil have a lower glycemic index and are beneficial to bone. These diets have a higher CQI and LCDS (as reported by Nouri and coauthors) and provide a rich source of antioxidants, vitamins, minerals, and other nutrients (such as calcium, magnesium, and vitamins B, C, and K), which are all beneficial to bone. Gao and others have reported that implementing a low glycemic index pulse-based diet (lentils, peas, beans) is superior to a regular hospital diet in preventing the increase in bone loss that typically occurs during hospitalization with enforced bed rest.

Most reports of the impact of carbohydrates on bone health are from observational studies. In an interventional study, Dalskov and coauthors randomly assigned children aged 5-18 years who had parents with overweight to one of five diets (high protein/low glycemic index, high protein/high glycemic index, low protein/low glycemic index, low protein/high glycemic index, or regular) for 6 months.

Contrasting with our understanding that protein intake is overall good for bone, this study found that among patients receiving a high–glycemic index diet, those who were on a high-protein diet had greater reductions in a bone formation marker than did those on a low-protein diet, with no major changes observed with the other diets. This suggests the influence of associated dietary nutrients on bone outcomes and that protein intake may modify the effects of dietary carbohydrates on bone formation. Similarly, the fat content of food can alter the glycemic index and thus may modify the impact of dietary carbohydrates on bone.

In summary, available data suggest that the quantity and quality of carbohydrates, including the glycemic index of food, may affect bone health and that it is important to exercise moderation in the consumption of such foods. However, there are only a few studies that have examined these associations, and more studies are necessary to further clarify the impact of dietary carbohydrates on bone as well as any modifications of these effects by other associated food groups. These studies will allow us to refine our recommendations to our patients as we advance our understanding of the impact of the combined effects of various dietary nutrients on bone.

Madhusmita Misra, MD, MPH, is chief of the division of pediatric endocrinology, Mass General for Children, Boston, and serves or has served as a director, officer, partner, employee, advisor, consultant, or trustee for AbbVie, Sanofi, and Ipsen.

A version of this article first appeared on Medscape.com.

Publications
Publications
Topics
Article Type
Sections
Disallow All Ads
Content Gating
No Gating (article Unlocked/Free)
Alternative CME
Disqus Comments
Default
Use ProPublica
Hide sidebar & use full width
render the right sidebar.
Conference Recap Checkbox
Not Conference Recap
Clinical Edge
Display the Slideshow in this Article
Medscape Article
Display survey writer
Reuters content
Disable Inline Native ads
WebMD Article

Robotic peritoneal vaginoplasty

Article Type
Changed
Fri, 03/10/2023 - 14:00

When the Food and Drug Administration first approved the da Vinci Surgical System (Intuitive Surgical, Sunnyvale, Calif.) for adult use in 2000, it altered the face of minimally invasive surgery across a multitude of specialties. Improved three-dimensional visualization and enhanced instrument articulation facilitates complex dissections and intracorporeal suturing. While the standard of care for gender-affirming vaginoplasty remains the single-stage penile inversion vaginoplasty, robotic procedures are quickly emerging as alternative options for both primary and revisional surgeries.

Dr. K. Ashley Brandt

The single-stage penile inversion vaginoplasty requires an adequate amount of penoscrotal tissue not only to line a neovaginal canal that measures 12-15 cm, but also to create external vulvar structures. While this is often sufficient in most candidates, there is an increasing number of patients who are receiving puberty blockers, resulting in penoscrotal hypoplasia.

Alternatively, there are patients who experience loss of vaginal depth and vaginal stenosis who seek revisional surgeries. Additional donor sites for skin grafting are available and include the lower abdomen and thighs, although patients may not want these donor site scars. With these donor sites, there is also concern about graft contracture, which could lead to recurrent vaginal stenosis.1 Robotic peritoneal vaginoplasty and robotic enteric vaginoplasty can serve as additional options for patients seeking revisional surgery or who have insufficient genital skin. One benefit of using peritoneal flaps is that they are hairless and are well vascularized with minimal donor site morbidity.1 Currently, there are two predominant techniques that utilize peritoneal flaps: the modified Davydov procedure and the tubularized urachus-peritoneal hinge flap.

The modified Davydov technique, which originated in the treatment of congenital vaginal agenesis in cisgender women, involves the creation of anterior and posterior peritoneal flaps. This type of peritoneal vaginoplasty is more commonly utilized for primary cases.

Ideally, there is a robotic surgeon (typically a urologist) working in tandem with the perineal surgeon. The robotic surgeon makes a horizontal incision along the peritoneal ridge at the rectovesical junction and continues the dissection within Denonvilliers fascia, between the prostate and rectum, to the pelvic floor. This dissection is like that performed in a robot-assisted laparoscopic prostatectomy.

Simultaneously, the perineal surgeon will break through the pelvic floor with assistance of the robotic view. Peritoneal flaps are raised from the anterior rectum and posterior bladder.2,3 In primary cases, the penoscrotal flap is introduced into the abdomen from the perineum and sutured to the anterior and posterior peritoneum to create a circumferential canal. At the apex of the neovagina, these anterior and posterior flaps are then sutured together.2,3

The tubularized urachus-peritoneal hinge flap technique is predominantly used for revision cases in patients who experienced neovaginal shortening and desire increased neovaginal depth. As peritoneal reach is limited, candidates for this procedure must have both adequate width and neovaginal canal depth.4 Once intra-abdominal access is achieved, an anterior peritoneal flap is mobilized to the level of the bladder and rotated 180 degrees inferiorly.4 The superior aspect of the flap is flipped is mobilized and is sutured to the peritoneum at the apex of the neovaginal canal.

The main benefit of these procedures, compared with traditional techniques, is increased neovaginal depth. The average vaginal length in patients undergoing peritoneal vaginoplasties is 14.2 cm, compared with 11.6 cm achieved in those using skin grafts.1,3 However, many surgeons report achieving 14-15 cm of depth with the traditional vaginoplasty. There are insufficient short- and long-term data for the peritoneal technique to recommend this as a first-line procedure.

Complications for peritoneal vaginoplasty procedures are similar to those of single-stage penile inversion vaginoplasty cases but with additional operative risks associated with laparoscopic/robotic surgery. These risks include injury to viscera and major vessels during initial intra-abdominal access, intra-abdominal adhesions, port site hernias, need to convert to an open procedure, and equipment malfunction.2 Additional postoperative risks include pelvic abscess formation, dehiscence of the peritoneal-vaginal incision, and peritoneal perforation during dilation.2,3 Surgeons and institutions must also weigh the cost of using the robot versus the cost of additional revisional surgical procedures. While initial studies evaluating robotic peritoneal vaginoplasty procedures have yielded promising preliminary results, additional studies are warranted.

Dr. Brandt is an ob.gyn. and fellowship-trained gender-affirming surgeon in West Reading, Pa.

References

1. Salibian AA et al. Plast Reconstr Surg. 2021;147(4):634e-43e.

2. Dy GW et al. In: Nikolavsky D and Blakely SA, eds. Urological care for the transgender patient: A comprehensive guide. Switzerland: Springer, 2021:237-48.

3. Jacoby A et al. J Urol. 2019;201(6):1171-5.

4. Smith SM et al. J Sex Med. 2022;10(6):100572.

Publications
Topics
Sections

When the Food and Drug Administration first approved the da Vinci Surgical System (Intuitive Surgical, Sunnyvale, Calif.) for adult use in 2000, it altered the face of minimally invasive surgery across a multitude of specialties. Improved three-dimensional visualization and enhanced instrument articulation facilitates complex dissections and intracorporeal suturing. While the standard of care for gender-affirming vaginoplasty remains the single-stage penile inversion vaginoplasty, robotic procedures are quickly emerging as alternative options for both primary and revisional surgeries.

Dr. K. Ashley Brandt

The single-stage penile inversion vaginoplasty requires an adequate amount of penoscrotal tissue not only to line a neovaginal canal that measures 12-15 cm, but also to create external vulvar structures. While this is often sufficient in most candidates, there is an increasing number of patients who are receiving puberty blockers, resulting in penoscrotal hypoplasia.

Alternatively, there are patients who experience loss of vaginal depth and vaginal stenosis who seek revisional surgeries. Additional donor sites for skin grafting are available and include the lower abdomen and thighs, although patients may not want these donor site scars. With these donor sites, there is also concern about graft contracture, which could lead to recurrent vaginal stenosis.1 Robotic peritoneal vaginoplasty and robotic enteric vaginoplasty can serve as additional options for patients seeking revisional surgery or who have insufficient genital skin. One benefit of using peritoneal flaps is that they are hairless and are well vascularized with minimal donor site morbidity.1 Currently, there are two predominant techniques that utilize peritoneal flaps: the modified Davydov procedure and the tubularized urachus-peritoneal hinge flap.

The modified Davydov technique, which originated in the treatment of congenital vaginal agenesis in cisgender women, involves the creation of anterior and posterior peritoneal flaps. This type of peritoneal vaginoplasty is more commonly utilized for primary cases.

Ideally, there is a robotic surgeon (typically a urologist) working in tandem with the perineal surgeon. The robotic surgeon makes a horizontal incision along the peritoneal ridge at the rectovesical junction and continues the dissection within Denonvilliers fascia, between the prostate and rectum, to the pelvic floor. This dissection is like that performed in a robot-assisted laparoscopic prostatectomy.

Simultaneously, the perineal surgeon will break through the pelvic floor with assistance of the robotic view. Peritoneal flaps are raised from the anterior rectum and posterior bladder.2,3 In primary cases, the penoscrotal flap is introduced into the abdomen from the perineum and sutured to the anterior and posterior peritoneum to create a circumferential canal. At the apex of the neovagina, these anterior and posterior flaps are then sutured together.2,3

The tubularized urachus-peritoneal hinge flap technique is predominantly used for revision cases in patients who experienced neovaginal shortening and desire increased neovaginal depth. As peritoneal reach is limited, candidates for this procedure must have both adequate width and neovaginal canal depth.4 Once intra-abdominal access is achieved, an anterior peritoneal flap is mobilized to the level of the bladder and rotated 180 degrees inferiorly.4 The superior aspect of the flap is flipped is mobilized and is sutured to the peritoneum at the apex of the neovaginal canal.

The main benefit of these procedures, compared with traditional techniques, is increased neovaginal depth. The average vaginal length in patients undergoing peritoneal vaginoplasties is 14.2 cm, compared with 11.6 cm achieved in those using skin grafts.1,3 However, many surgeons report achieving 14-15 cm of depth with the traditional vaginoplasty. There are insufficient short- and long-term data for the peritoneal technique to recommend this as a first-line procedure.

Complications for peritoneal vaginoplasty procedures are similar to those of single-stage penile inversion vaginoplasty cases but with additional operative risks associated with laparoscopic/robotic surgery. These risks include injury to viscera and major vessels during initial intra-abdominal access, intra-abdominal adhesions, port site hernias, need to convert to an open procedure, and equipment malfunction.2 Additional postoperative risks include pelvic abscess formation, dehiscence of the peritoneal-vaginal incision, and peritoneal perforation during dilation.2,3 Surgeons and institutions must also weigh the cost of using the robot versus the cost of additional revisional surgical procedures. While initial studies evaluating robotic peritoneal vaginoplasty procedures have yielded promising preliminary results, additional studies are warranted.

Dr. Brandt is an ob.gyn. and fellowship-trained gender-affirming surgeon in West Reading, Pa.

References

1. Salibian AA et al. Plast Reconstr Surg. 2021;147(4):634e-43e.

2. Dy GW et al. In: Nikolavsky D and Blakely SA, eds. Urological care for the transgender patient: A comprehensive guide. Switzerland: Springer, 2021:237-48.

3. Jacoby A et al. J Urol. 2019;201(6):1171-5.

4. Smith SM et al. J Sex Med. 2022;10(6):100572.

When the Food and Drug Administration first approved the da Vinci Surgical System (Intuitive Surgical, Sunnyvale, Calif.) for adult use in 2000, it altered the face of minimally invasive surgery across a multitude of specialties. Improved three-dimensional visualization and enhanced instrument articulation facilitates complex dissections and intracorporeal suturing. While the standard of care for gender-affirming vaginoplasty remains the single-stage penile inversion vaginoplasty, robotic procedures are quickly emerging as alternative options for both primary and revisional surgeries.

Dr. K. Ashley Brandt

The single-stage penile inversion vaginoplasty requires an adequate amount of penoscrotal tissue not only to line a neovaginal canal that measures 12-15 cm, but also to create external vulvar structures. While this is often sufficient in most candidates, there is an increasing number of patients who are receiving puberty blockers, resulting in penoscrotal hypoplasia.

Alternatively, there are patients who experience loss of vaginal depth and vaginal stenosis who seek revisional surgeries. Additional donor sites for skin grafting are available and include the lower abdomen and thighs, although patients may not want these donor site scars. With these donor sites, there is also concern about graft contracture, which could lead to recurrent vaginal stenosis.1 Robotic peritoneal vaginoplasty and robotic enteric vaginoplasty can serve as additional options for patients seeking revisional surgery or who have insufficient genital skin. One benefit of using peritoneal flaps is that they are hairless and are well vascularized with minimal donor site morbidity.1 Currently, there are two predominant techniques that utilize peritoneal flaps: the modified Davydov procedure and the tubularized urachus-peritoneal hinge flap.

The modified Davydov technique, which originated in the treatment of congenital vaginal agenesis in cisgender women, involves the creation of anterior and posterior peritoneal flaps. This type of peritoneal vaginoplasty is more commonly utilized for primary cases.

Ideally, there is a robotic surgeon (typically a urologist) working in tandem with the perineal surgeon. The robotic surgeon makes a horizontal incision along the peritoneal ridge at the rectovesical junction and continues the dissection within Denonvilliers fascia, between the prostate and rectum, to the pelvic floor. This dissection is like that performed in a robot-assisted laparoscopic prostatectomy.

Simultaneously, the perineal surgeon will break through the pelvic floor with assistance of the robotic view. Peritoneal flaps are raised from the anterior rectum and posterior bladder.2,3 In primary cases, the penoscrotal flap is introduced into the abdomen from the perineum and sutured to the anterior and posterior peritoneum to create a circumferential canal. At the apex of the neovagina, these anterior and posterior flaps are then sutured together.2,3

The tubularized urachus-peritoneal hinge flap technique is predominantly used for revision cases in patients who experienced neovaginal shortening and desire increased neovaginal depth. As peritoneal reach is limited, candidates for this procedure must have both adequate width and neovaginal canal depth.4 Once intra-abdominal access is achieved, an anterior peritoneal flap is mobilized to the level of the bladder and rotated 180 degrees inferiorly.4 The superior aspect of the flap is flipped is mobilized and is sutured to the peritoneum at the apex of the neovaginal canal.

The main benefit of these procedures, compared with traditional techniques, is increased neovaginal depth. The average vaginal length in patients undergoing peritoneal vaginoplasties is 14.2 cm, compared with 11.6 cm achieved in those using skin grafts.1,3 However, many surgeons report achieving 14-15 cm of depth with the traditional vaginoplasty. There are insufficient short- and long-term data for the peritoneal technique to recommend this as a first-line procedure.

Complications for peritoneal vaginoplasty procedures are similar to those of single-stage penile inversion vaginoplasty cases but with additional operative risks associated with laparoscopic/robotic surgery. These risks include injury to viscera and major vessels during initial intra-abdominal access, intra-abdominal adhesions, port site hernias, need to convert to an open procedure, and equipment malfunction.2 Additional postoperative risks include pelvic abscess formation, dehiscence of the peritoneal-vaginal incision, and peritoneal perforation during dilation.2,3 Surgeons and institutions must also weigh the cost of using the robot versus the cost of additional revisional surgical procedures. While initial studies evaluating robotic peritoneal vaginoplasty procedures have yielded promising preliminary results, additional studies are warranted.

Dr. Brandt is an ob.gyn. and fellowship-trained gender-affirming surgeon in West Reading, Pa.

References

1. Salibian AA et al. Plast Reconstr Surg. 2021;147(4):634e-43e.

2. Dy GW et al. In: Nikolavsky D and Blakely SA, eds. Urological care for the transgender patient: A comprehensive guide. Switzerland: Springer, 2021:237-48.

3. Jacoby A et al. J Urol. 2019;201(6):1171-5.

4. Smith SM et al. J Sex Med. 2022;10(6):100572.

Publications
Publications
Topics
Article Type
Sections
Disallow All Ads
Content Gating
No Gating (article Unlocked/Free)
Alternative CME
Disqus Comments
Default
Use ProPublica
Hide sidebar & use full width
render the right sidebar.
Conference Recap Checkbox
Not Conference Recap
Clinical Edge
Display the Slideshow in this Article
Medscape Article
Display survey writer
Reuters content
Disable Inline Native ads
WebMD Article

U.S. vs. French guidelines for osteoporosis treatment

Article Type
Changed
Fri, 03/03/2023 - 14:03

The American College of Physicians has just updated its guidelines for osteoporosis treatment. Bernard Cortet, MD, PhD, chairperson of the Osteoporosis Research and Information Group and head of the rheumatology department at Lille (France) University Hospital, has agreed to compare the new U.S. guidelines to the 2018 French recommendations written under the aegis of the French Society for Rheumatology and GRIO. Dr. Cortet participated in drafting the French recommendations.

Question: The ACP “strongly” recommends initial pharmacologic treatment with bisphosphonate antiresorptive drugs (alendronate, ibandronate, risedronate, zoledronate) in postmenopausal females diagnosed with primary osteoporosis. Isn’t this what the SFR–GRIO have been recommending for many years?

Answer: The ACP reinforces its stance by arguing that in postmenopausal females with primary osteoporosis, bisphosphonates have the most favorable balance between benefits, harms, patient values and preferences, and cost among the drug classes that were evaluated. In addition to net clinical benefits, bisphosphonates are much cheaper than other pharmacologic treatments and are available in generic oral and injectable formulations.

Our French recommendations specify the choice of drug based on the type of fracture in women and on their bone mineral density (BMD). However, bisphosphonates are definitely given pride of place. When treatment for osteoporosis needs to be started, most of the time, a bisphosphonate is the treatment of choice.

Nevertheless, as also highlighted by the ACP, a more “aggressive” approach must be considered for more severe cases.

In the case of a severe fracture, the French recommendations indicate that all treatments can be prescribed. However, zoledronic acid should be favored as first-line treatment for a hip fracture. In other cases – with or without a nonsevere fracture – the therapeutic indication depends on the BMD values, and in difficult cases, on tools such as FRAX [the Fracture Risk Assessment Tool].

Our guidance strongly recommends opting for an injection in other contexts, such as significant decrease in bone density, presence of comorbidities, poor treatment compliance, brain function disorders, and polymedication.

Q. But it’s not really as simple as prescribing a bisphosphonate, is it?

A. You’re right, many people find the idea of taking bisphosphonates worrying because of associated jaw problems – osteonecrosis of the jaw – or atypical femoral fractures, based on what they’ve read on the Internet, where these serious adverse events are on display front and center with no mention of how often they actually happen and, often, failing to mention how effective bisphosphonates truly are.

These complications are real, but fortunately rare, especially during the first 5 years of treatment. To put this into context, for bisphosphonates, there’s one case of osteonecrosis of the jaw for every 10,000. And for denosumab, there are five cases for every 10,000. For atypical fractures, there’s one case for every 30,000 to 50,000.

Q. The U.S. guidelines also recommend that clinicians use a RANK ligand inhibitor – denosumab, also an antiresorptive drug – as second-line medical treatment. This is to reduce the risk of fractures in postmenopausal women diagnosed with primary osteoporosis and presenting with contraindications or side effects of bisphosphonates. Do you support the use of denosumab as second-line treatment?

A. French legislation classifies it as a second-line treatment, after bisphosphonates. However, there are arguments in favor of prescribing it as first-line treatment in some contexts. If denosumab is to be prescribed – via a twice-yearly subcutaneous injection – full compliance must be observed. If a patient is to stop taking denosumab, an opinion from a medical professional is required before treatment can be discontinued, and then treatment with bisphosphonates must be prescribed.

Q. The ACP recommends that clinicians use either a sclerostin inhibitor – romosozumab – or recombinant human parathyroid hormone – teriparatide – two anabolic agents, followed by a bisphosphonate, with the aim of reducing the risk of fractures. This is only used in women with primary osteoporosis who are at a very high risk of fracture. As romosozumab is not available in France, it’s not really worth discussing its use. Does this strategy seem advisable to you, though?

A. The main issue is what is understood by “women at a very high risk of fracture.” There’s no consensus on the definition of what constitutes a woman at a very high risk of fracture, but we can assume that it involves the combination of low BMD and at least one severe fracture.

The role of anabolic bone treatment, as [the ACP] has defined it, seems logical to me, because in cases of severe osteoporosis with fracture, the risk of recurrence is very high in the next 2-3 years. In a study comparing risedronate and teriparatide in cases of severe osteoporosis, teriparatide was more effective in reducing the recurrence of vertebral fractures.

The favorable opinion of the French National Authority for Health in relation to medical coverage for romosozumab in the treatment of severe postmenopausal osteoporosis in women under the age of 75 years with a history of severe fractures, a T-score less than –2.5, and no previous history of coronary artery disease dates to 2021. This is because medical coverage for this specific group was not listed in the marketing authorization (MA) description for this drug.

But the review by the Economic Committee for Health Products failed to reach a consensus regarding the price. Today, in theory, romosozumab can be dispensed in France by hospital pharmacies, because it is approved for use in public hospitals. Romosozumab is a very interesting drug for relatively young women, especially those with multiple vertebral fractures. This injectable treatment is more effective than teriparatide in increasing BMD values and more effective than alendronate in preventing the recurrence of fractures.

Regarding medical coverage, as it stands, in cases where patients have a T-score less than or equal to –3, the 2018 SFR–GRIO recommends starting treatment even if the patient has no fractures. In cases with severe fractures combined with very low BMD (T-score ≤ –3), injectable treatments may be used to reach a bone density target (T-score > –2.5 to –2 for the hip) at the end of the treatment plan. [These treatments include] zoledronic acid, denosumab (in case of bisphosphonate failure or intolerance), or a treatment plan with teriparatide (covered by medical insurance if the patient has at least two vertebral fractures) followed by an antiresorptive drug (bisphosphonate or denosumab).

Romosozumab is a humanized monoclonal antibody (IgG2) that binds to sclerostin and acts as an inhibitor. This increases bone formation because of the activation of [bone lining cells], the production of bone matrix by osteoblasts, and the recruitment of osteochondroprogenitor cells. Moreover, romosozumab causes changes in the expression of osteoclast mediators, which decreases bone resorption. Together, these two effects that increase bone formation and decrease bone resorption lead to the rapid increase of trabecular and cortical bone mass, as well as improvements in bone structure and strength.

Women treated with a bone anabolic agent must take an antiresorptive agent at the end of their treatment so that the benefits from the treatment remain in the long term. The French and U.S. guidelines line up on this point.

In patients with two prevalent vertebral fractures, the U.S. guidelines state that teriparatide can be prescribed as first-line treatment at diagnosis in the absence of any contraindications. We agree on this point as well.

Moreover, in women under the age of 70 years with osteoporosis requiring treatment, French experts recommend prescribing raloxifene, a selective estrogen-receptor modulator. This is if the risk of nonvertebral fracture is low, as defined by the absence of the following criteria: low hip T-score, risk of falling, and history of nonvertebral fracture. Opportunities for its use are limited, and it doesn’t even figure among the U.S. recommendations.

Q. The ACP recommends that clinicians adopt an individualized approach regarding whether to start medical treatment with a bisphosphonate in women over age 65 years with low bone mass (osteopenia) to reduce the risk of fractures. If treatment is started, they›re of the opinion that a bisphosphonate must be used. What are the recommendations in France?

A. It should be noted that this recommendation by the ACP is conditional because of the low-certainty evidence.

Here’s a brief reminder of important things to note: a T-score between –2.5 and –1 indicates osteopenia; a T-score less than or equal to –2.5 indicates osteoporosis; a T-score less than or equal to –2.5 with one or several fractures indicates severe osteoporosis. The French recommendations state that treatment is not justified if a patient’s T-score is higher than –2 and there’s no presence of fractures, even with risk factors (and/or multiple falls). For T-scores less than or equal to –2 and higher than –3, the decision to prescribe depends on the specialist.

Q. The ACP recommends that clinicians use bisphosphonates for the initial medical treatment to reduce the risk of fractures in men diagnosed with primary osteoporosis.

A. The ACP recommends that clinicians use a RANK ligand inhibitor – denosumab – as second-line medical treatment to reduce the risk of fractures in men diagnosed with primary osteoporosis who present with contraindications or who are experiencing side effects of bisphosphonates. This treatment is not covered by health insurance for men in France.

Between 20% and 25% of clinical osteoporotic fractures occur in men. After age 50 years, men are roughly 20% more likely to experience an osteoporotic fracture in their lifetime. The French recommendations regarding the management and treatment of osteoporosis in men were published in 2021.

In the case of severe fractures (vertebrae, pelvis, upper end of the femur, distal femur, proximal humerus) attributable to bone fragility, osteoporosis treatment is recommended if one of the T-scores is less than or equal to –1.

In the case of nonsevere fractures (particularly wrist and ankle) attributable to bone fragility, osteoporosis treatment is recommended if one of the T-scores is less than or equal to –2. If there are no fractures, osteoporosis treatment is recommended in men at risk of bone fragility or of falling and if one of the T-scores is less than or equal to –3. In patients who had a fracture of the upper end of the femur attributable to bone fragility, zoledronic acid is recommended as first-line treatment.

For men with a severe nonvertebral fracture, single vertebral fracture, or nonsevere fracture, two treatments are indicated and covered by health insurance in France: zoledronic acid and risedronate. In men with at least two vertebral fractures, the following treatments are indicated and covered by health insurance in France: teriparatide and risedronate. In this case, teriparatide is prescribed for a period of 18 months. It must be followed by a prescription of oral or intravenous bisphosphonates.

Q. What is your take on the HAS update to the proper use of osteoporosis medication that’s just been published?

A. Like in the 2018 SFR–GRIO guidelines, no update has been made to the section on postmenopausal osteoporosis, except for the HAS introduction to the proper use of romosozumab, even though it’s not covered by health insurance in France.

In accordance with the MA, it doesn’t make sense to include this drug on the list of treatment options available for women with and without fractures, as it’s not included in the HAS-selected list of drugs covered by health insurance in France.

But I’m glad that the HAS has adopted the GRIO and SFR recommendations regarding corticosteroid-induced osteoporosis. Preventive treatment for corticosteroid-induced osteoporosis must be considered as soon as the daily dose of corticosteroids reaches or exceeds the equivalent of 7.5 mg of prednisone and when the estimated duration of corticosteroid therapy exceeds 3 months.

In summary, in women and men over the age of 50 years, the intake of the equivalent of 7.5 mg/day or more of prednisone or a history of a low-trauma fracture or being age 70 years or older, even with a T-score less than or equal to –2.5 for one of the two sites, indicates prescribing a bisphosphonate. Teriparatide is indicated if the patient has two vertebral fractures.

This article was translated from Medscape’s French edition.

A version of this article first appeared on Medscape.com.

Publications
Topics
Sections

The American College of Physicians has just updated its guidelines for osteoporosis treatment. Bernard Cortet, MD, PhD, chairperson of the Osteoporosis Research and Information Group and head of the rheumatology department at Lille (France) University Hospital, has agreed to compare the new U.S. guidelines to the 2018 French recommendations written under the aegis of the French Society for Rheumatology and GRIO. Dr. Cortet participated in drafting the French recommendations.

Question: The ACP “strongly” recommends initial pharmacologic treatment with bisphosphonate antiresorptive drugs (alendronate, ibandronate, risedronate, zoledronate) in postmenopausal females diagnosed with primary osteoporosis. Isn’t this what the SFR–GRIO have been recommending for many years?

Answer: The ACP reinforces its stance by arguing that in postmenopausal females with primary osteoporosis, bisphosphonates have the most favorable balance between benefits, harms, patient values and preferences, and cost among the drug classes that were evaluated. In addition to net clinical benefits, bisphosphonates are much cheaper than other pharmacologic treatments and are available in generic oral and injectable formulations.

Our French recommendations specify the choice of drug based on the type of fracture in women and on their bone mineral density (BMD). However, bisphosphonates are definitely given pride of place. When treatment for osteoporosis needs to be started, most of the time, a bisphosphonate is the treatment of choice.

Nevertheless, as also highlighted by the ACP, a more “aggressive” approach must be considered for more severe cases.

In the case of a severe fracture, the French recommendations indicate that all treatments can be prescribed. However, zoledronic acid should be favored as first-line treatment for a hip fracture. In other cases – with or without a nonsevere fracture – the therapeutic indication depends on the BMD values, and in difficult cases, on tools such as FRAX [the Fracture Risk Assessment Tool].

Our guidance strongly recommends opting for an injection in other contexts, such as significant decrease in bone density, presence of comorbidities, poor treatment compliance, brain function disorders, and polymedication.

Q. But it’s not really as simple as prescribing a bisphosphonate, is it?

A. You’re right, many people find the idea of taking bisphosphonates worrying because of associated jaw problems – osteonecrosis of the jaw – or atypical femoral fractures, based on what they’ve read on the Internet, where these serious adverse events are on display front and center with no mention of how often they actually happen and, often, failing to mention how effective bisphosphonates truly are.

These complications are real, but fortunately rare, especially during the first 5 years of treatment. To put this into context, for bisphosphonates, there’s one case of osteonecrosis of the jaw for every 10,000. And for denosumab, there are five cases for every 10,000. For atypical fractures, there’s one case for every 30,000 to 50,000.

Q. The U.S. guidelines also recommend that clinicians use a RANK ligand inhibitor – denosumab, also an antiresorptive drug – as second-line medical treatment. This is to reduce the risk of fractures in postmenopausal women diagnosed with primary osteoporosis and presenting with contraindications or side effects of bisphosphonates. Do you support the use of denosumab as second-line treatment?

A. French legislation classifies it as a second-line treatment, after bisphosphonates. However, there are arguments in favor of prescribing it as first-line treatment in some contexts. If denosumab is to be prescribed – via a twice-yearly subcutaneous injection – full compliance must be observed. If a patient is to stop taking denosumab, an opinion from a medical professional is required before treatment can be discontinued, and then treatment with bisphosphonates must be prescribed.

Q. The ACP recommends that clinicians use either a sclerostin inhibitor – romosozumab – or recombinant human parathyroid hormone – teriparatide – two anabolic agents, followed by a bisphosphonate, with the aim of reducing the risk of fractures. This is only used in women with primary osteoporosis who are at a very high risk of fracture. As romosozumab is not available in France, it’s not really worth discussing its use. Does this strategy seem advisable to you, though?

A. The main issue is what is understood by “women at a very high risk of fracture.” There’s no consensus on the definition of what constitutes a woman at a very high risk of fracture, but we can assume that it involves the combination of low BMD and at least one severe fracture.

The role of anabolic bone treatment, as [the ACP] has defined it, seems logical to me, because in cases of severe osteoporosis with fracture, the risk of recurrence is very high in the next 2-3 years. In a study comparing risedronate and teriparatide in cases of severe osteoporosis, teriparatide was more effective in reducing the recurrence of vertebral fractures.

The favorable opinion of the French National Authority for Health in relation to medical coverage for romosozumab in the treatment of severe postmenopausal osteoporosis in women under the age of 75 years with a history of severe fractures, a T-score less than –2.5, and no previous history of coronary artery disease dates to 2021. This is because medical coverage for this specific group was not listed in the marketing authorization (MA) description for this drug.

But the review by the Economic Committee for Health Products failed to reach a consensus regarding the price. Today, in theory, romosozumab can be dispensed in France by hospital pharmacies, because it is approved for use in public hospitals. Romosozumab is a very interesting drug for relatively young women, especially those with multiple vertebral fractures. This injectable treatment is more effective than teriparatide in increasing BMD values and more effective than alendronate in preventing the recurrence of fractures.

Regarding medical coverage, as it stands, in cases where patients have a T-score less than or equal to –3, the 2018 SFR–GRIO recommends starting treatment even if the patient has no fractures. In cases with severe fractures combined with very low BMD (T-score ≤ –3), injectable treatments may be used to reach a bone density target (T-score > –2.5 to –2 for the hip) at the end of the treatment plan. [These treatments include] zoledronic acid, denosumab (in case of bisphosphonate failure or intolerance), or a treatment plan with teriparatide (covered by medical insurance if the patient has at least two vertebral fractures) followed by an antiresorptive drug (bisphosphonate or denosumab).

Romosozumab is a humanized monoclonal antibody (IgG2) that binds to sclerostin and acts as an inhibitor. This increases bone formation because of the activation of [bone lining cells], the production of bone matrix by osteoblasts, and the recruitment of osteochondroprogenitor cells. Moreover, romosozumab causes changes in the expression of osteoclast mediators, which decreases bone resorption. Together, these two effects that increase bone formation and decrease bone resorption lead to the rapid increase of trabecular and cortical bone mass, as well as improvements in bone structure and strength.

Women treated with a bone anabolic agent must take an antiresorptive agent at the end of their treatment so that the benefits from the treatment remain in the long term. The French and U.S. guidelines line up on this point.

In patients with two prevalent vertebral fractures, the U.S. guidelines state that teriparatide can be prescribed as first-line treatment at diagnosis in the absence of any contraindications. We agree on this point as well.

Moreover, in women under the age of 70 years with osteoporosis requiring treatment, French experts recommend prescribing raloxifene, a selective estrogen-receptor modulator. This is if the risk of nonvertebral fracture is low, as defined by the absence of the following criteria: low hip T-score, risk of falling, and history of nonvertebral fracture. Opportunities for its use are limited, and it doesn’t even figure among the U.S. recommendations.

Q. The ACP recommends that clinicians adopt an individualized approach regarding whether to start medical treatment with a bisphosphonate in women over age 65 years with low bone mass (osteopenia) to reduce the risk of fractures. If treatment is started, they›re of the opinion that a bisphosphonate must be used. What are the recommendations in France?

A. It should be noted that this recommendation by the ACP is conditional because of the low-certainty evidence.

Here’s a brief reminder of important things to note: a T-score between –2.5 and –1 indicates osteopenia; a T-score less than or equal to –2.5 indicates osteoporosis; a T-score less than or equal to –2.5 with one or several fractures indicates severe osteoporosis. The French recommendations state that treatment is not justified if a patient’s T-score is higher than –2 and there’s no presence of fractures, even with risk factors (and/or multiple falls). For T-scores less than or equal to –2 and higher than –3, the decision to prescribe depends on the specialist.

Q. The ACP recommends that clinicians use bisphosphonates for the initial medical treatment to reduce the risk of fractures in men diagnosed with primary osteoporosis.

A. The ACP recommends that clinicians use a RANK ligand inhibitor – denosumab – as second-line medical treatment to reduce the risk of fractures in men diagnosed with primary osteoporosis who present with contraindications or who are experiencing side effects of bisphosphonates. This treatment is not covered by health insurance for men in France.

Between 20% and 25% of clinical osteoporotic fractures occur in men. After age 50 years, men are roughly 20% more likely to experience an osteoporotic fracture in their lifetime. The French recommendations regarding the management and treatment of osteoporosis in men were published in 2021.

In the case of severe fractures (vertebrae, pelvis, upper end of the femur, distal femur, proximal humerus) attributable to bone fragility, osteoporosis treatment is recommended if one of the T-scores is less than or equal to –1.

In the case of nonsevere fractures (particularly wrist and ankle) attributable to bone fragility, osteoporosis treatment is recommended if one of the T-scores is less than or equal to –2. If there are no fractures, osteoporosis treatment is recommended in men at risk of bone fragility or of falling and if one of the T-scores is less than or equal to –3. In patients who had a fracture of the upper end of the femur attributable to bone fragility, zoledronic acid is recommended as first-line treatment.

For men with a severe nonvertebral fracture, single vertebral fracture, or nonsevere fracture, two treatments are indicated and covered by health insurance in France: zoledronic acid and risedronate. In men with at least two vertebral fractures, the following treatments are indicated and covered by health insurance in France: teriparatide and risedronate. In this case, teriparatide is prescribed for a period of 18 months. It must be followed by a prescription of oral or intravenous bisphosphonates.

Q. What is your take on the HAS update to the proper use of osteoporosis medication that’s just been published?

A. Like in the 2018 SFR–GRIO guidelines, no update has been made to the section on postmenopausal osteoporosis, except for the HAS introduction to the proper use of romosozumab, even though it’s not covered by health insurance in France.

In accordance with the MA, it doesn’t make sense to include this drug on the list of treatment options available for women with and without fractures, as it’s not included in the HAS-selected list of drugs covered by health insurance in France.

But I’m glad that the HAS has adopted the GRIO and SFR recommendations regarding corticosteroid-induced osteoporosis. Preventive treatment for corticosteroid-induced osteoporosis must be considered as soon as the daily dose of corticosteroids reaches or exceeds the equivalent of 7.5 mg of prednisone and when the estimated duration of corticosteroid therapy exceeds 3 months.

In summary, in women and men over the age of 50 years, the intake of the equivalent of 7.5 mg/day or more of prednisone or a history of a low-trauma fracture or being age 70 years or older, even with a T-score less than or equal to –2.5 for one of the two sites, indicates prescribing a bisphosphonate. Teriparatide is indicated if the patient has two vertebral fractures.

This article was translated from Medscape’s French edition.

A version of this article first appeared on Medscape.com.

The American College of Physicians has just updated its guidelines for osteoporosis treatment. Bernard Cortet, MD, PhD, chairperson of the Osteoporosis Research and Information Group and head of the rheumatology department at Lille (France) University Hospital, has agreed to compare the new U.S. guidelines to the 2018 French recommendations written under the aegis of the French Society for Rheumatology and GRIO. Dr. Cortet participated in drafting the French recommendations.

Question: The ACP “strongly” recommends initial pharmacologic treatment with bisphosphonate antiresorptive drugs (alendronate, ibandronate, risedronate, zoledronate) in postmenopausal females diagnosed with primary osteoporosis. Isn’t this what the SFR–GRIO have been recommending for many years?

Answer: The ACP reinforces its stance by arguing that in postmenopausal females with primary osteoporosis, bisphosphonates have the most favorable balance between benefits, harms, patient values and preferences, and cost among the drug classes that were evaluated. In addition to net clinical benefits, bisphosphonates are much cheaper than other pharmacologic treatments and are available in generic oral and injectable formulations.

Our French recommendations specify the choice of drug based on the type of fracture in women and on their bone mineral density (BMD). However, bisphosphonates are definitely given pride of place. When treatment for osteoporosis needs to be started, most of the time, a bisphosphonate is the treatment of choice.

Nevertheless, as also highlighted by the ACP, a more “aggressive” approach must be considered for more severe cases.

In the case of a severe fracture, the French recommendations indicate that all treatments can be prescribed. However, zoledronic acid should be favored as first-line treatment for a hip fracture. In other cases – with or without a nonsevere fracture – the therapeutic indication depends on the BMD values, and in difficult cases, on tools such as FRAX [the Fracture Risk Assessment Tool].

Our guidance strongly recommends opting for an injection in other contexts, such as significant decrease in bone density, presence of comorbidities, poor treatment compliance, brain function disorders, and polymedication.

Q. But it’s not really as simple as prescribing a bisphosphonate, is it?

A. You’re right, many people find the idea of taking bisphosphonates worrying because of associated jaw problems – osteonecrosis of the jaw – or atypical femoral fractures, based on what they’ve read on the Internet, where these serious adverse events are on display front and center with no mention of how often they actually happen and, often, failing to mention how effective bisphosphonates truly are.

These complications are real, but fortunately rare, especially during the first 5 years of treatment. To put this into context, for bisphosphonates, there’s one case of osteonecrosis of the jaw for every 10,000. And for denosumab, there are five cases for every 10,000. For atypical fractures, there’s one case for every 30,000 to 50,000.

Q. The U.S. guidelines also recommend that clinicians use a RANK ligand inhibitor – denosumab, also an antiresorptive drug – as second-line medical treatment. This is to reduce the risk of fractures in postmenopausal women diagnosed with primary osteoporosis and presenting with contraindications or side effects of bisphosphonates. Do you support the use of denosumab as second-line treatment?

A. French legislation classifies it as a second-line treatment, after bisphosphonates. However, there are arguments in favor of prescribing it as first-line treatment in some contexts. If denosumab is to be prescribed – via a twice-yearly subcutaneous injection – full compliance must be observed. If a patient is to stop taking denosumab, an opinion from a medical professional is required before treatment can be discontinued, and then treatment with bisphosphonates must be prescribed.

Q. The ACP recommends that clinicians use either a sclerostin inhibitor – romosozumab – or recombinant human parathyroid hormone – teriparatide – two anabolic agents, followed by a bisphosphonate, with the aim of reducing the risk of fractures. This is only used in women with primary osteoporosis who are at a very high risk of fracture. As romosozumab is not available in France, it’s not really worth discussing its use. Does this strategy seem advisable to you, though?

A. The main issue is what is understood by “women at a very high risk of fracture.” There’s no consensus on the definition of what constitutes a woman at a very high risk of fracture, but we can assume that it involves the combination of low BMD and at least one severe fracture.

The role of anabolic bone treatment, as [the ACP] has defined it, seems logical to me, because in cases of severe osteoporosis with fracture, the risk of recurrence is very high in the next 2-3 years. In a study comparing risedronate and teriparatide in cases of severe osteoporosis, teriparatide was more effective in reducing the recurrence of vertebral fractures.

The favorable opinion of the French National Authority for Health in relation to medical coverage for romosozumab in the treatment of severe postmenopausal osteoporosis in women under the age of 75 years with a history of severe fractures, a T-score less than –2.5, and no previous history of coronary artery disease dates to 2021. This is because medical coverage for this specific group was not listed in the marketing authorization (MA) description for this drug.

But the review by the Economic Committee for Health Products failed to reach a consensus regarding the price. Today, in theory, romosozumab can be dispensed in France by hospital pharmacies, because it is approved for use in public hospitals. Romosozumab is a very interesting drug for relatively young women, especially those with multiple vertebral fractures. This injectable treatment is more effective than teriparatide in increasing BMD values and more effective than alendronate in preventing the recurrence of fractures.

Regarding medical coverage, as it stands, in cases where patients have a T-score less than or equal to –3, the 2018 SFR–GRIO recommends starting treatment even if the patient has no fractures. In cases with severe fractures combined with very low BMD (T-score ≤ –3), injectable treatments may be used to reach a bone density target (T-score > –2.5 to –2 for the hip) at the end of the treatment plan. [These treatments include] zoledronic acid, denosumab (in case of bisphosphonate failure or intolerance), or a treatment plan with teriparatide (covered by medical insurance if the patient has at least two vertebral fractures) followed by an antiresorptive drug (bisphosphonate or denosumab).

Romosozumab is a humanized monoclonal antibody (IgG2) that binds to sclerostin and acts as an inhibitor. This increases bone formation because of the activation of [bone lining cells], the production of bone matrix by osteoblasts, and the recruitment of osteochondroprogenitor cells. Moreover, romosozumab causes changes in the expression of osteoclast mediators, which decreases bone resorption. Together, these two effects that increase bone formation and decrease bone resorption lead to the rapid increase of trabecular and cortical bone mass, as well as improvements in bone structure and strength.

Women treated with a bone anabolic agent must take an antiresorptive agent at the end of their treatment so that the benefits from the treatment remain in the long term. The French and U.S. guidelines line up on this point.

In patients with two prevalent vertebral fractures, the U.S. guidelines state that teriparatide can be prescribed as first-line treatment at diagnosis in the absence of any contraindications. We agree on this point as well.

Moreover, in women under the age of 70 years with osteoporosis requiring treatment, French experts recommend prescribing raloxifene, a selective estrogen-receptor modulator. This is if the risk of nonvertebral fracture is low, as defined by the absence of the following criteria: low hip T-score, risk of falling, and history of nonvertebral fracture. Opportunities for its use are limited, and it doesn’t even figure among the U.S. recommendations.

Q. The ACP recommends that clinicians adopt an individualized approach regarding whether to start medical treatment with a bisphosphonate in women over age 65 years with low bone mass (osteopenia) to reduce the risk of fractures. If treatment is started, they›re of the opinion that a bisphosphonate must be used. What are the recommendations in France?

A. It should be noted that this recommendation by the ACP is conditional because of the low-certainty evidence.

Here’s a brief reminder of important things to note: a T-score between –2.5 and –1 indicates osteopenia; a T-score less than or equal to –2.5 indicates osteoporosis; a T-score less than or equal to –2.5 with one or several fractures indicates severe osteoporosis. The French recommendations state that treatment is not justified if a patient’s T-score is higher than –2 and there’s no presence of fractures, even with risk factors (and/or multiple falls). For T-scores less than or equal to –2 and higher than –3, the decision to prescribe depends on the specialist.

Q. The ACP recommends that clinicians use bisphosphonates for the initial medical treatment to reduce the risk of fractures in men diagnosed with primary osteoporosis.

A. The ACP recommends that clinicians use a RANK ligand inhibitor – denosumab – as second-line medical treatment to reduce the risk of fractures in men diagnosed with primary osteoporosis who present with contraindications or who are experiencing side effects of bisphosphonates. This treatment is not covered by health insurance for men in France.

Between 20% and 25% of clinical osteoporotic fractures occur in men. After age 50 years, men are roughly 20% more likely to experience an osteoporotic fracture in their lifetime. The French recommendations regarding the management and treatment of osteoporosis in men were published in 2021.

In the case of severe fractures (vertebrae, pelvis, upper end of the femur, distal femur, proximal humerus) attributable to bone fragility, osteoporosis treatment is recommended if one of the T-scores is less than or equal to –1.

In the case of nonsevere fractures (particularly wrist and ankle) attributable to bone fragility, osteoporosis treatment is recommended if one of the T-scores is less than or equal to –2. If there are no fractures, osteoporosis treatment is recommended in men at risk of bone fragility or of falling and if one of the T-scores is less than or equal to –3. In patients who had a fracture of the upper end of the femur attributable to bone fragility, zoledronic acid is recommended as first-line treatment.

For men with a severe nonvertebral fracture, single vertebral fracture, or nonsevere fracture, two treatments are indicated and covered by health insurance in France: zoledronic acid and risedronate. In men with at least two vertebral fractures, the following treatments are indicated and covered by health insurance in France: teriparatide and risedronate. In this case, teriparatide is prescribed for a period of 18 months. It must be followed by a prescription of oral or intravenous bisphosphonates.

Q. What is your take on the HAS update to the proper use of osteoporosis medication that’s just been published?

A. Like in the 2018 SFR–GRIO guidelines, no update has been made to the section on postmenopausal osteoporosis, except for the HAS introduction to the proper use of romosozumab, even though it’s not covered by health insurance in France.

In accordance with the MA, it doesn’t make sense to include this drug on the list of treatment options available for women with and without fractures, as it’s not included in the HAS-selected list of drugs covered by health insurance in France.

But I’m glad that the HAS has adopted the GRIO and SFR recommendations regarding corticosteroid-induced osteoporosis. Preventive treatment for corticosteroid-induced osteoporosis must be considered as soon as the daily dose of corticosteroids reaches or exceeds the equivalent of 7.5 mg of prednisone and when the estimated duration of corticosteroid therapy exceeds 3 months.

In summary, in women and men over the age of 50 years, the intake of the equivalent of 7.5 mg/day or more of prednisone or a history of a low-trauma fracture or being age 70 years or older, even with a T-score less than or equal to –2.5 for one of the two sites, indicates prescribing a bisphosphonate. Teriparatide is indicated if the patient has two vertebral fractures.

This article was translated from Medscape’s French edition.

A version of this article first appeared on Medscape.com.

Publications
Publications
Topics
Article Type
Sections
Disallow All Ads
Content Gating
No Gating (article Unlocked/Free)
Alternative CME
Disqus Comments
Default
Use ProPublica
Hide sidebar & use full width
render the right sidebar.
Conference Recap Checkbox
Not Conference Recap
Clinical Edge
Display the Slideshow in this Article
Medscape Article
Display survey writer
Reuters content
Disable Inline Native ads
WebMD Article

Back pain: Red flags and when to image

Article Type
Changed
Tue, 03/14/2023 - 17:46

 

This transcript has been edited for clarity.

Matthew F. Watto, MD: Welcome back to The Curbsiders. On tonight’s episode, we are going to be talking about back pain. This is based on an interview, Back Pain Update with Dr. Austin Baraki. He gave us some great pearls about how to manage back pain, which we see so much of in primary care. I’ll use one of my famous teaching techniques: If the patient has any kind of back pain, they should just not move. Right?

Paul N. Williams, MD: That’s right, Matt – we should recommend bedrest until they get better for anyone who has any back pain? No. For back pain, early activity and exercise are great. Patients are often concerned that physical therapy will make their pain worse, so they don’t exercise. This misunderstanding is not surprising. They believe that if they are experiencing pain, it’s facilitating more damage, which is not necessarily the case. It will get better, and a little bit of anticipatory guidance goes a long way in terms of managing patient expectations related to early mobilization, early exercise, and physical therapy.

Dr. Watto: Absolutely. One of the goals of treatment is symptom relief to the extent that we’re able to achieve. We’re not expecting the pain to go to zero. That just doesn’t happen, especially if someone’s on a medication long term. Another goal is return to function. We want them sleeping. We want them to be able to tolerate movement.

We have medications – NSAIDs and muscle relaxants, which are actually tranquilizers. But most therapy for back pain doesn’t involve medications. It involves active movement, so we have to find movement that the patient enjoys doing. Passive treatments, things being done to patients, just don’t work as well.
 

Dr. Williams: We should be clear – we’re talking primarily about chronic back pain here. For acute back pain, we actually have some decent medications, but acute back pain tends to improve no matter what you do. We don’t have much to offer pharmacologically for chronic low back pain. The best modalities usually involve physical activity of some kind.

Dr. Watto: Let’s discuss the evaluation of back pain. Something that always comes up: Should we order imaging, and is there a right time to get it? Dr. Baraki was very clear about when to do imaging. Two big buckets of patients might need imaging.

First, a patient who has a serious underlying condition and you’re using imaging to try to diagnose it; or in a chronic setting, a patient who needs surgery, and imaging is part of the presurgical evaluation. We talked about red flags.

The red flags are major trauma, where we have reason to believe there might be something going on – if we strongly suspect infection, or the patient is injecting drugs. If the patient has a history of cancer, we would be worried that they might have a recurrence. Those are some of the main red flags. With a patient who has osteoporosis or is on chronic steroids, you might even be able to get by with plain films instead of an MRI to look for fracture.

The other thing I wanted to ask you about is, when should we get imaging? Are there any pitfalls we need to worry about?
 

Dr. Williams: I always like podcasts I’m not on because I enjoy listening to them much more. Dr. Baraki talked about the very specific language that is used in radiology reports, such as spondylitis, spondylolysis, and multilevel degenerative disease. They sound bad, but if they are just reframed as age-related degenerative changes, that sounds so much more benign. When discussing with patients, we should avoid medical jargon and say that we saw some changes that we would expect for someone of your age. That sounds so much better than saying we saw multilevel degenerative disease, which sounds like an alarming pathology if you’re not a physician. Without being inaccurate, we should frame the discussion such that we aren’t providing a very specific diagnosis, because that is rarely the case with chronic low back pain. Typically, many things are going on and you may never identify a single unifying diagnosis, which doesn’t tend to help anyway.

Dr. Watto: There’s evidence showing that if the radiology report uses clinical terminology that both clinician and patient think of as less serious, they are less likely to proceed to more invasive treatments. Calling an episode of back pain a “lumbar strain” helps the patient understand that this is a pretty common thing. Almost everyone is going to have an episode of back pain at some point in their life, and almost all of them will get better. Most of the time there’s no serious underlying condition.

This was a great discussion with Dr. Baraki. Click on Back Pain Update with Dr Austin Baraki to hear the full discussion. Until next time, I’ve been Dr. Matthew Frank Watto.
 

Dr. Williams: And I’m Dr. Paul Nelson Williams.

Dr. Watto is Clinical Assistant Professor, Department of Medicine, University of Pennsylvania, Philadelphia. Dr. Williams is Associate Professor of Clinical Medicine, Department of General Internal Medicine, Temple University, Philadelphia. Neither reported any conflicts of interest.

A version of this article first appeared on Medscape.com.

Publications
Topics
Sections

 

This transcript has been edited for clarity.

Matthew F. Watto, MD: Welcome back to The Curbsiders. On tonight’s episode, we are going to be talking about back pain. This is based on an interview, Back Pain Update with Dr. Austin Baraki. He gave us some great pearls about how to manage back pain, which we see so much of in primary care. I’ll use one of my famous teaching techniques: If the patient has any kind of back pain, they should just not move. Right?

Paul N. Williams, MD: That’s right, Matt – we should recommend bedrest until they get better for anyone who has any back pain? No. For back pain, early activity and exercise are great. Patients are often concerned that physical therapy will make their pain worse, so they don’t exercise. This misunderstanding is not surprising. They believe that if they are experiencing pain, it’s facilitating more damage, which is not necessarily the case. It will get better, and a little bit of anticipatory guidance goes a long way in terms of managing patient expectations related to early mobilization, early exercise, and physical therapy.

Dr. Watto: Absolutely. One of the goals of treatment is symptom relief to the extent that we’re able to achieve. We’re not expecting the pain to go to zero. That just doesn’t happen, especially if someone’s on a medication long term. Another goal is return to function. We want them sleeping. We want them to be able to tolerate movement.

We have medications – NSAIDs and muscle relaxants, which are actually tranquilizers. But most therapy for back pain doesn’t involve medications. It involves active movement, so we have to find movement that the patient enjoys doing. Passive treatments, things being done to patients, just don’t work as well.
 

Dr. Williams: We should be clear – we’re talking primarily about chronic back pain here. For acute back pain, we actually have some decent medications, but acute back pain tends to improve no matter what you do. We don’t have much to offer pharmacologically for chronic low back pain. The best modalities usually involve physical activity of some kind.

Dr. Watto: Let’s discuss the evaluation of back pain. Something that always comes up: Should we order imaging, and is there a right time to get it? Dr. Baraki was very clear about when to do imaging. Two big buckets of patients might need imaging.

First, a patient who has a serious underlying condition and you’re using imaging to try to diagnose it; or in a chronic setting, a patient who needs surgery, and imaging is part of the presurgical evaluation. We talked about red flags.

The red flags are major trauma, where we have reason to believe there might be something going on – if we strongly suspect infection, or the patient is injecting drugs. If the patient has a history of cancer, we would be worried that they might have a recurrence. Those are some of the main red flags. With a patient who has osteoporosis or is on chronic steroids, you might even be able to get by with plain films instead of an MRI to look for fracture.

The other thing I wanted to ask you about is, when should we get imaging? Are there any pitfalls we need to worry about?
 

Dr. Williams: I always like podcasts I’m not on because I enjoy listening to them much more. Dr. Baraki talked about the very specific language that is used in radiology reports, such as spondylitis, spondylolysis, and multilevel degenerative disease. They sound bad, but if they are just reframed as age-related degenerative changes, that sounds so much more benign. When discussing with patients, we should avoid medical jargon and say that we saw some changes that we would expect for someone of your age. That sounds so much better than saying we saw multilevel degenerative disease, which sounds like an alarming pathology if you’re not a physician. Without being inaccurate, we should frame the discussion such that we aren’t providing a very specific diagnosis, because that is rarely the case with chronic low back pain. Typically, many things are going on and you may never identify a single unifying diagnosis, which doesn’t tend to help anyway.

Dr. Watto: There’s evidence showing that if the radiology report uses clinical terminology that both clinician and patient think of as less serious, they are less likely to proceed to more invasive treatments. Calling an episode of back pain a “lumbar strain” helps the patient understand that this is a pretty common thing. Almost everyone is going to have an episode of back pain at some point in their life, and almost all of them will get better. Most of the time there’s no serious underlying condition.

This was a great discussion with Dr. Baraki. Click on Back Pain Update with Dr Austin Baraki to hear the full discussion. Until next time, I’ve been Dr. Matthew Frank Watto.
 

Dr. Williams: And I’m Dr. Paul Nelson Williams.

Dr. Watto is Clinical Assistant Professor, Department of Medicine, University of Pennsylvania, Philadelphia. Dr. Williams is Associate Professor of Clinical Medicine, Department of General Internal Medicine, Temple University, Philadelphia. Neither reported any conflicts of interest.

A version of this article first appeared on Medscape.com.

 

This transcript has been edited for clarity.

Matthew F. Watto, MD: Welcome back to The Curbsiders. On tonight’s episode, we are going to be talking about back pain. This is based on an interview, Back Pain Update with Dr. Austin Baraki. He gave us some great pearls about how to manage back pain, which we see so much of in primary care. I’ll use one of my famous teaching techniques: If the patient has any kind of back pain, they should just not move. Right?

Paul N. Williams, MD: That’s right, Matt – we should recommend bedrest until they get better for anyone who has any back pain? No. For back pain, early activity and exercise are great. Patients are often concerned that physical therapy will make their pain worse, so they don’t exercise. This misunderstanding is not surprising. They believe that if they are experiencing pain, it’s facilitating more damage, which is not necessarily the case. It will get better, and a little bit of anticipatory guidance goes a long way in terms of managing patient expectations related to early mobilization, early exercise, and physical therapy.

Dr. Watto: Absolutely. One of the goals of treatment is symptom relief to the extent that we’re able to achieve. We’re not expecting the pain to go to zero. That just doesn’t happen, especially if someone’s on a medication long term. Another goal is return to function. We want them sleeping. We want them to be able to tolerate movement.

We have medications – NSAIDs and muscle relaxants, which are actually tranquilizers. But most therapy for back pain doesn’t involve medications. It involves active movement, so we have to find movement that the patient enjoys doing. Passive treatments, things being done to patients, just don’t work as well.
 

Dr. Williams: We should be clear – we’re talking primarily about chronic back pain here. For acute back pain, we actually have some decent medications, but acute back pain tends to improve no matter what you do. We don’t have much to offer pharmacologically for chronic low back pain. The best modalities usually involve physical activity of some kind.

Dr. Watto: Let’s discuss the evaluation of back pain. Something that always comes up: Should we order imaging, and is there a right time to get it? Dr. Baraki was very clear about when to do imaging. Two big buckets of patients might need imaging.

First, a patient who has a serious underlying condition and you’re using imaging to try to diagnose it; or in a chronic setting, a patient who needs surgery, and imaging is part of the presurgical evaluation. We talked about red flags.

The red flags are major trauma, where we have reason to believe there might be something going on – if we strongly suspect infection, or the patient is injecting drugs. If the patient has a history of cancer, we would be worried that they might have a recurrence. Those are some of the main red flags. With a patient who has osteoporosis or is on chronic steroids, you might even be able to get by with plain films instead of an MRI to look for fracture.

The other thing I wanted to ask you about is, when should we get imaging? Are there any pitfalls we need to worry about?
 

Dr. Williams: I always like podcasts I’m not on because I enjoy listening to them much more. Dr. Baraki talked about the very specific language that is used in radiology reports, such as spondylitis, spondylolysis, and multilevel degenerative disease. They sound bad, but if they are just reframed as age-related degenerative changes, that sounds so much more benign. When discussing with patients, we should avoid medical jargon and say that we saw some changes that we would expect for someone of your age. That sounds so much better than saying we saw multilevel degenerative disease, which sounds like an alarming pathology if you’re not a physician. Without being inaccurate, we should frame the discussion such that we aren’t providing a very specific diagnosis, because that is rarely the case with chronic low back pain. Typically, many things are going on and you may never identify a single unifying diagnosis, which doesn’t tend to help anyway.

Dr. Watto: There’s evidence showing that if the radiology report uses clinical terminology that both clinician and patient think of as less serious, they are less likely to proceed to more invasive treatments. Calling an episode of back pain a “lumbar strain” helps the patient understand that this is a pretty common thing. Almost everyone is going to have an episode of back pain at some point in their life, and almost all of them will get better. Most of the time there’s no serious underlying condition.

This was a great discussion with Dr. Baraki. Click on Back Pain Update with Dr Austin Baraki to hear the full discussion. Until next time, I’ve been Dr. Matthew Frank Watto.
 

Dr. Williams: And I’m Dr. Paul Nelson Williams.

Dr. Watto is Clinical Assistant Professor, Department of Medicine, University of Pennsylvania, Philadelphia. Dr. Williams is Associate Professor of Clinical Medicine, Department of General Internal Medicine, Temple University, Philadelphia. Neither reported any conflicts of interest.

A version of this article first appeared on Medscape.com.

Publications
Publications
Topics
Article Type
Sections
Disallow All Ads
Content Gating
No Gating (article Unlocked/Free)
Alternative CME
Disqus Comments
Default
Use ProPublica
Hide sidebar & use full width
render the right sidebar.
Conference Recap Checkbox
Not Conference Recap
Clinical Edge
Display the Slideshow in this Article
Medscape Article
Display survey writer
Reuters content
Disable Inline Native ads
WebMD Article

ESMO guidelines provide ‘clear blueprint’ for managing immunotherapy toxicities

Article Type
Changed
Wed, 03/01/2023 - 11:09

 

This transcript has been edited for clarity.

I’m David Kerr, professor of cancer medicine at the University of Oxford. I’d like to talk to you today about something specific and generic around guidelines.

Annals of Oncology, my old journal, has just published an outstanding set of guidelines delivered by the ESMO (European Society for Medical Oncology) guidelines group. It’s around the management of toxicities from immunotherapy, and it’s the ESMO Clinical Practice Guideline for diagnosis, treatment, and follow-up, delivered by Dr. Haanen and, of course, a number of colleagues on behalf of the wider committee.

Have a look at it. I’m not going to talk about the details of it. It’s very well written. It’s very clear and evidence based, of course. There are many helpful hints and a very clear blueprint as to how we should better manage the myriad of potential side effects from immunotherapy.

It tells us a little about the basis of the science, some of the mechanistic work that’s going on in allowing us to understand why some people react in such different ways, almost as if the immune systems are primed to overreact. It gives a very helpful, stepwise look at how we best diagnose, manage, and, in the longer term, follow up patients who have problems with these very important drugs.

All of us recognize the extraordinary impact they’ve made across a wide range of different tumor types, and therefore, as practicing oncologists and health care professionals in the field, all of us need to understand better the details as to how we better care for our patients on these drugs.

Have a look at it. It’s well written and useful, and I think it’s a document that I’ll turn to when I’m looking for a refresher or advice in the future.

The generic focus is about guidelines. Many years ago, I was one of the architects of the British National Cancer Plan, and for me, there were four simple principles at that stage in our development of how we would improve the delivery of cancer control in the United Kingdom. It was around site specialization, particularly of our surgical colleagues who embraced this with vigor. God bless them.

It was using guidelines to help level up the quality of treatment that we were giving, of course underpinned by research, and using – one would hope – modern IT and telecommunications to improve the networking that we use to deliver multidisciplinary cancer care, one of the key elements. Guidelines were embedded in that.

A couple of years ago, we did a survey of cancer physicians around the world. Almost 30 different countries were represented, and we asked which guidelines were most used. It was a very interesting set of responses. The three dominant guidelines – this will surprise no one – are the NCCN (National Comprehensive Cancer Network) guidelines, the ESMO guidelines, and the ASCO (American Society of Clinical Oncology) guidelines.

Rather than selecting one and one being completely dominant, what seemed to be the case is that our colleagues around the world dipped in and used all three. They may prefer NCCN for some particular tumor type or some particular aspect of how they’re structured, but at the same time, we would dip into the ESMO guidelines for specific bits of help, as well as the ASCO guidelines.

I find this fascinating. I assume that in different regions, depending on how they were affiliated in terms of additional training or links to Europe or links to the United States, that one or other of these guideline groups would predominate, but no. In each country, in each region, given the large data bank that we have of guidelines now, it’s a sort of pick-and-mix situation.

I was initially surprised but then took comfort from it. There’s nothing I hate more than the wasted energy of reduplication and saying, well come on, if there is one guideline set that does truly command the attention of the world, then the other should stop. It’s wasted energy, which is something that none of us can afford.

The fact that each of these trusted, evidence-based, beautifully presented guidelines is used in different ways was important. A message to the guideline groups from me is: “Thank you for your professionalism, for the hard work of hundreds of cancer specialists from all different specialties, and for their contribution to developing these guidelines.”

It’s worth it, it’s working, people are using them, and they’re making a difference. It’s all about leveling up the quality of cancer care that we deliver.

Specifically, have a look at the ESMO immune guidelines. They are great. I hope you find them helpful. Generically, thanks to all of you who are contributing and working so hard to make these data available to improve the quality of cancer care around the world.

Thanks for listening, as always. I’m interested in any comments that you might have, but for the time being, Medscapers, ahoy.

David J. Kerr, CBE, MD, DSc, is a professor of cancer medicine at the University of Oxford. He reported conflicts of interest with Celleron Therapeutics, Oxford Cancer Biomarkers, Afrox, GlaxoSmithKline, Bayer, Genomic Health, and Merck Serono.

A version of this article first appeared on Medscape.com.

Publications
Topics
Sections

 

This transcript has been edited for clarity.

I’m David Kerr, professor of cancer medicine at the University of Oxford. I’d like to talk to you today about something specific and generic around guidelines.

Annals of Oncology, my old journal, has just published an outstanding set of guidelines delivered by the ESMO (European Society for Medical Oncology) guidelines group. It’s around the management of toxicities from immunotherapy, and it’s the ESMO Clinical Practice Guideline for diagnosis, treatment, and follow-up, delivered by Dr. Haanen and, of course, a number of colleagues on behalf of the wider committee.

Have a look at it. I’m not going to talk about the details of it. It’s very well written. It’s very clear and evidence based, of course. There are many helpful hints and a very clear blueprint as to how we should better manage the myriad of potential side effects from immunotherapy.

It tells us a little about the basis of the science, some of the mechanistic work that’s going on in allowing us to understand why some people react in such different ways, almost as if the immune systems are primed to overreact. It gives a very helpful, stepwise look at how we best diagnose, manage, and, in the longer term, follow up patients who have problems with these very important drugs.

All of us recognize the extraordinary impact they’ve made across a wide range of different tumor types, and therefore, as practicing oncologists and health care professionals in the field, all of us need to understand better the details as to how we better care for our patients on these drugs.

Have a look at it. It’s well written and useful, and I think it’s a document that I’ll turn to when I’m looking for a refresher or advice in the future.

The generic focus is about guidelines. Many years ago, I was one of the architects of the British National Cancer Plan, and for me, there were four simple principles at that stage in our development of how we would improve the delivery of cancer control in the United Kingdom. It was around site specialization, particularly of our surgical colleagues who embraced this with vigor. God bless them.

It was using guidelines to help level up the quality of treatment that we were giving, of course underpinned by research, and using – one would hope – modern IT and telecommunications to improve the networking that we use to deliver multidisciplinary cancer care, one of the key elements. Guidelines were embedded in that.

A couple of years ago, we did a survey of cancer physicians around the world. Almost 30 different countries were represented, and we asked which guidelines were most used. It was a very interesting set of responses. The three dominant guidelines – this will surprise no one – are the NCCN (National Comprehensive Cancer Network) guidelines, the ESMO guidelines, and the ASCO (American Society of Clinical Oncology) guidelines.

Rather than selecting one and one being completely dominant, what seemed to be the case is that our colleagues around the world dipped in and used all three. They may prefer NCCN for some particular tumor type or some particular aspect of how they’re structured, but at the same time, we would dip into the ESMO guidelines for specific bits of help, as well as the ASCO guidelines.

I find this fascinating. I assume that in different regions, depending on how they were affiliated in terms of additional training or links to Europe or links to the United States, that one or other of these guideline groups would predominate, but no. In each country, in each region, given the large data bank that we have of guidelines now, it’s a sort of pick-and-mix situation.

I was initially surprised but then took comfort from it. There’s nothing I hate more than the wasted energy of reduplication and saying, well come on, if there is one guideline set that does truly command the attention of the world, then the other should stop. It’s wasted energy, which is something that none of us can afford.

The fact that each of these trusted, evidence-based, beautifully presented guidelines is used in different ways was important. A message to the guideline groups from me is: “Thank you for your professionalism, for the hard work of hundreds of cancer specialists from all different specialties, and for their contribution to developing these guidelines.”

It’s worth it, it’s working, people are using them, and they’re making a difference. It’s all about leveling up the quality of cancer care that we deliver.

Specifically, have a look at the ESMO immune guidelines. They are great. I hope you find them helpful. Generically, thanks to all of you who are contributing and working so hard to make these data available to improve the quality of cancer care around the world.

Thanks for listening, as always. I’m interested in any comments that you might have, but for the time being, Medscapers, ahoy.

David J. Kerr, CBE, MD, DSc, is a professor of cancer medicine at the University of Oxford. He reported conflicts of interest with Celleron Therapeutics, Oxford Cancer Biomarkers, Afrox, GlaxoSmithKline, Bayer, Genomic Health, and Merck Serono.

A version of this article first appeared on Medscape.com.

 

This transcript has been edited for clarity.

I’m David Kerr, professor of cancer medicine at the University of Oxford. I’d like to talk to you today about something specific and generic around guidelines.

Annals of Oncology, my old journal, has just published an outstanding set of guidelines delivered by the ESMO (European Society for Medical Oncology) guidelines group. It’s around the management of toxicities from immunotherapy, and it’s the ESMO Clinical Practice Guideline for diagnosis, treatment, and follow-up, delivered by Dr. Haanen and, of course, a number of colleagues on behalf of the wider committee.

Have a look at it. I’m not going to talk about the details of it. It’s very well written. It’s very clear and evidence based, of course. There are many helpful hints and a very clear blueprint as to how we should better manage the myriad of potential side effects from immunotherapy.

It tells us a little about the basis of the science, some of the mechanistic work that’s going on in allowing us to understand why some people react in such different ways, almost as if the immune systems are primed to overreact. It gives a very helpful, stepwise look at how we best diagnose, manage, and, in the longer term, follow up patients who have problems with these very important drugs.

All of us recognize the extraordinary impact they’ve made across a wide range of different tumor types, and therefore, as practicing oncologists and health care professionals in the field, all of us need to understand better the details as to how we better care for our patients on these drugs.

Have a look at it. It’s well written and useful, and I think it’s a document that I’ll turn to when I’m looking for a refresher or advice in the future.

The generic focus is about guidelines. Many years ago, I was one of the architects of the British National Cancer Plan, and for me, there were four simple principles at that stage in our development of how we would improve the delivery of cancer control in the United Kingdom. It was around site specialization, particularly of our surgical colleagues who embraced this with vigor. God bless them.

It was using guidelines to help level up the quality of treatment that we were giving, of course underpinned by research, and using – one would hope – modern IT and telecommunications to improve the networking that we use to deliver multidisciplinary cancer care, one of the key elements. Guidelines were embedded in that.

A couple of years ago, we did a survey of cancer physicians around the world. Almost 30 different countries were represented, and we asked which guidelines were most used. It was a very interesting set of responses. The three dominant guidelines – this will surprise no one – are the NCCN (National Comprehensive Cancer Network) guidelines, the ESMO guidelines, and the ASCO (American Society of Clinical Oncology) guidelines.

Rather than selecting one and one being completely dominant, what seemed to be the case is that our colleagues around the world dipped in and used all three. They may prefer NCCN for some particular tumor type or some particular aspect of how they’re structured, but at the same time, we would dip into the ESMO guidelines for specific bits of help, as well as the ASCO guidelines.

I find this fascinating. I assume that in different regions, depending on how they were affiliated in terms of additional training or links to Europe or links to the United States, that one or other of these guideline groups would predominate, but no. In each country, in each region, given the large data bank that we have of guidelines now, it’s a sort of pick-and-mix situation.

I was initially surprised but then took comfort from it. There’s nothing I hate more than the wasted energy of reduplication and saying, well come on, if there is one guideline set that does truly command the attention of the world, then the other should stop. It’s wasted energy, which is something that none of us can afford.

The fact that each of these trusted, evidence-based, beautifully presented guidelines is used in different ways was important. A message to the guideline groups from me is: “Thank you for your professionalism, for the hard work of hundreds of cancer specialists from all different specialties, and for their contribution to developing these guidelines.”

It’s worth it, it’s working, people are using them, and they’re making a difference. It’s all about leveling up the quality of cancer care that we deliver.

Specifically, have a look at the ESMO immune guidelines. They are great. I hope you find them helpful. Generically, thanks to all of you who are contributing and working so hard to make these data available to improve the quality of cancer care around the world.

Thanks for listening, as always. I’m interested in any comments that you might have, but for the time being, Medscapers, ahoy.

David J. Kerr, CBE, MD, DSc, is a professor of cancer medicine at the University of Oxford. He reported conflicts of interest with Celleron Therapeutics, Oxford Cancer Biomarkers, Afrox, GlaxoSmithKline, Bayer, Genomic Health, and Merck Serono.

A version of this article first appeared on Medscape.com.

Publications
Publications
Topics
Article Type
Sections
Disallow All Ads
Content Gating
No Gating (article Unlocked/Free)
Alternative CME
Disqus Comments
Default
Use ProPublica
Hide sidebar & use full width
render the right sidebar.
Conference Recap Checkbox
Not Conference Recap
Clinical Edge
Display the Slideshow in this Article
Medscape Article
Display survey writer
Reuters content
Disable Inline Native ads
WebMD Article

Reimagining psychiatric assessment and interventions as procedures

Article Type
Changed
Wed, 03/01/2023 - 01:15
Display Headline
Reimagining psychiatric assessment and interventions as procedures

Many psychiatric physicians lament the dearth of procedures in psychiatry compared to other medical specialties such as surgery, cardiology, gastroenterology, or radiology. The few procedures in psychiatry include electroconvulsive therapy (ECT), repetitive transcranial magnetic stimulation, and vagus nerve stimulation, which are restricted to a small number of sites and not available for most psychiatric practitioners. This lack of tangible/physical procedures should not be surprising because psychiatry deals with disorders of the mind, which are invisible.

However, when one closely examines what psychiatrists do in daily practice to heal our patients, most of what we do actually qualifies as “procedures” although no hardware, machines, or gadgets are involved. Treating psychiatric brain disorders (aka mental illness) requires exquisite skills and expertise, just like medical specialties that use machines to measure or treat various body organs.

It’s time to relabel psychiatric interventions as procedures designed to improve anomalous thoughts, affect, emotions, cognition, and behavior. After giving it some thought (and with a bit of tongue in cheek), I came up with the following list of “psychiatric procedures”:

  • Psychosocial exploratory laparotomy: The comprehensive psychiatric assessment and mental status exam.
  • Chemotherapy: Oral or injective pharmacotherapeutic intervention.
  • Psychoplastic repair: Neuroplasticity, including neurogenesis, synaptogenesis, and dendritic spine regeneration, have been shown to be associated with both psychotherapy and psychotropic medications.1,2
  • Suicidectomy: Extracting the lethal urge to die by suicide.
  • Anger debridement: Removing the irritability and destructive anger outbursts frequently associated with various psychopathologies.
  • Anxiety ablation: Eliminating the noxious emotional state of anxiety and frightening panic attacks.
  • Empathy infusion: Enabling patients to become more understanding of other people and bolstering their impaired “theory of mind.”
  • Personality transplant: Replacing a maladaptive personality with a healthier one (eg, using dialectical behavior therapy for borderline personality disorder).
  • Cognitive LASIK: To improve insight, analogous to how ophthalmologic LASIK improves sight.
  • Mental embolectomy: Removing a blockage to repair rigid attitudes and develop “open-mindedness.”
  • Behavioral dilation and curettage (D&C): To rid patients of negative attributes such as impulsivity or reckless behavior.
  • Psychotherapeutic anesthesia: Numbing emotional pain or severe grief reaction.
  • Social anastomosis: Helping patients who are schizoid or isolative via group therapy, an effective interpersonal and social procedure.
  • Psychotherapeutic stent: To open the vessels of narrow-mindedness.
  • Cortico-psychological resuscitation (CPR): For patients experiencing stress-induced behavioral arrhythmias or emotional infarction.
  • Immunotherapy: Using various neuroprotective psychotropic medications with anti-inflammatory properties or employing evidence-based psychotherapy such as cognitive-behavior therapy (aka neuropsychotherapy), which have been shown to reduce inflammatory biomarkers such as C-reactive protein and cytokines.3
  • Psychotherapy: A neuromodulation procedure for a variety of psychiatric disorders.4
  • Neurobiological facelift: It is well established that neurogenesis, synaptogenesis, and dendritic spine sprouting are significantly increased with both neuroprotective psychotropic medications (antidepressants, lithium, valproate, and second-generationantipsychotics5) as well as with psych­otherapy. There is growing evidence of “premature brain aging” in schizophrenia, bipolar disorder, and depression, with shrinkage in the volume of the cortex and subcortical regions, especially the hippocampus. Psychiatric biopsychosocial interven­tion rebuilds those brain regions by stimulating and replenishing the neuropil and neuro­genic regions (dentate gyrus and subventricular zone). This is like performing virtual plastic surgery on a wrinkled brain and its sagging mind. MRI scans before and after ECT show a remarkable ≥10% increase in the volume of the hippocampus and amygdala, which translates to billions of new neurons, glia, and synapses.6

Reinventing psychiatric therapies as procedures may elicit sarcasm from skeptics, but when you think about it, it is justified. Excising depression is like excising a tumor, not with a scalpel, but virtually. Stabilizing the broken brain and mind after a psychotic episode (aka brain attack) is like stabilizing the heart after a myocardial infarction (aka heart attack). Just because the mind is virtual doesn’t mean it is not “real and tangible.” A desktop computer is visible, but the software that brings it to life is invisible. Healing the human mind requires multiple medical interventions by psychiatrists in hospitals and clinics, just like surgeons and endoscopists or cardiologists. Mental health care is as much procedural as other medical and surgical specialties.

One more thing: the validated clinical rating scales for various psychiatric brain disorders (eg, the Positive and Negative Syndrome Scale for schizophrenia, Montgomery-Åsberg Depression Rating Scale for depression, Young Mania Rating Scale for bipolar mania, Hamilton Anxiety Rating Scale for anxiety, Yale-Brown Obsessive Compulsive Scale for obsessive-compulsive disorder) are actual measurement procedures for the severity of the illness, just as a sphygmomanometer measures blood pressure and its improvement with treatment. There are also multiple cognitive test batteries to measure cognitive impairment.7

Finally, unlike psychiatric reimbursement, which is tethered to time, procedures are compensated more generously, irrespective of the time involved. The complexities of diagnosing and treating psychiatric brain disorders that dangerously disrupt thoughts, feelings, behavior, and cognition are just as intricate and demanding as the diagnosis and treatment of general medical and surgical conditions. They should all be equally appreciated as vital life-saving procedures for the human body, brain, and mind.

References

1. Nasrallah HA, Hopkins T, Pixley SK. Differential effects of antipsychotic and antidepressant drugs on neurogenic regions in rats. Brain Res. 2010;1354:23-29.

2. Tomasino B, Fabbro F. Increases in the right dorsolateral prefrontal cortex and decreases the rostral prefrontal cortex activation after-8 weeks of focused attention based mindfulness meditation. Brain Cogn. 2016;102:46-54.

3. Nasrallah HA. Repositioning psychotherapy as a neurobiological intervention. Current Psychiatry. 2013;12(12):18-19.

4. Nasrallah HA. Optimal psychiatric treatment: Target the brain and avoid the body. Current Psychiatry. 2022;21(12):3-6.

5. Chen AT, Nasrallah HA. Neuroprotective effects of the second generation antipsychotics. Schizophr Res. 2019;208:1-7.

6. Gryglewski G, Lanzenberger R, Silberbauer LR, et al. Meta-analysis of brain structural changes after electroconvulsive therapy in depression. Brain Stimul. 2021;14(4):927-937.

7. Nasrallah HA. The Cognition Self-Assessment Rating Scale for patients with schizophrenia. Current Psychiatry. 2023;22(3):30-34.

Article PDF
Author and Disclosure Information

Henry A. Nasrallah, MD, DLFAPA
Editor-in-Chief

Issue
Current Psychiatry - 22(3)
Publications
Page Number
4-5,12
Sections
Author and Disclosure Information

Henry A. Nasrallah, MD, DLFAPA
Editor-in-Chief

Author and Disclosure Information

Henry A. Nasrallah, MD, DLFAPA
Editor-in-Chief

Article PDF
Article PDF

Many psychiatric physicians lament the dearth of procedures in psychiatry compared to other medical specialties such as surgery, cardiology, gastroenterology, or radiology. The few procedures in psychiatry include electroconvulsive therapy (ECT), repetitive transcranial magnetic stimulation, and vagus nerve stimulation, which are restricted to a small number of sites and not available for most psychiatric practitioners. This lack of tangible/physical procedures should not be surprising because psychiatry deals with disorders of the mind, which are invisible.

However, when one closely examines what psychiatrists do in daily practice to heal our patients, most of what we do actually qualifies as “procedures” although no hardware, machines, or gadgets are involved. Treating psychiatric brain disorders (aka mental illness) requires exquisite skills and expertise, just like medical specialties that use machines to measure or treat various body organs.

It’s time to relabel psychiatric interventions as procedures designed to improve anomalous thoughts, affect, emotions, cognition, and behavior. After giving it some thought (and with a bit of tongue in cheek), I came up with the following list of “psychiatric procedures”:

  • Psychosocial exploratory laparotomy: The comprehensive psychiatric assessment and mental status exam.
  • Chemotherapy: Oral or injective pharmacotherapeutic intervention.
  • Psychoplastic repair: Neuroplasticity, including neurogenesis, synaptogenesis, and dendritic spine regeneration, have been shown to be associated with both psychotherapy and psychotropic medications.1,2
  • Suicidectomy: Extracting the lethal urge to die by suicide.
  • Anger debridement: Removing the irritability and destructive anger outbursts frequently associated with various psychopathologies.
  • Anxiety ablation: Eliminating the noxious emotional state of anxiety and frightening panic attacks.
  • Empathy infusion: Enabling patients to become more understanding of other people and bolstering their impaired “theory of mind.”
  • Personality transplant: Replacing a maladaptive personality with a healthier one (eg, using dialectical behavior therapy for borderline personality disorder).
  • Cognitive LASIK: To improve insight, analogous to how ophthalmologic LASIK improves sight.
  • Mental embolectomy: Removing a blockage to repair rigid attitudes and develop “open-mindedness.”
  • Behavioral dilation and curettage (D&C): To rid patients of negative attributes such as impulsivity or reckless behavior.
  • Psychotherapeutic anesthesia: Numbing emotional pain or severe grief reaction.
  • Social anastomosis: Helping patients who are schizoid or isolative via group therapy, an effective interpersonal and social procedure.
  • Psychotherapeutic stent: To open the vessels of narrow-mindedness.
  • Cortico-psychological resuscitation (CPR): For patients experiencing stress-induced behavioral arrhythmias or emotional infarction.
  • Immunotherapy: Using various neuroprotective psychotropic medications with anti-inflammatory properties or employing evidence-based psychotherapy such as cognitive-behavior therapy (aka neuropsychotherapy), which have been shown to reduce inflammatory biomarkers such as C-reactive protein and cytokines.3
  • Psychotherapy: A neuromodulation procedure for a variety of psychiatric disorders.4
  • Neurobiological facelift: It is well established that neurogenesis, synaptogenesis, and dendritic spine sprouting are significantly increased with both neuroprotective psychotropic medications (antidepressants, lithium, valproate, and second-generationantipsychotics5) as well as with psych­otherapy. There is growing evidence of “premature brain aging” in schizophrenia, bipolar disorder, and depression, with shrinkage in the volume of the cortex and subcortical regions, especially the hippocampus. Psychiatric biopsychosocial interven­tion rebuilds those brain regions by stimulating and replenishing the neuropil and neuro­genic regions (dentate gyrus and subventricular zone). This is like performing virtual plastic surgery on a wrinkled brain and its sagging mind. MRI scans before and after ECT show a remarkable ≥10% increase in the volume of the hippocampus and amygdala, which translates to billions of new neurons, glia, and synapses.6

Reinventing psychiatric therapies as procedures may elicit sarcasm from skeptics, but when you think about it, it is justified. Excising depression is like excising a tumor, not with a scalpel, but virtually. Stabilizing the broken brain and mind after a psychotic episode (aka brain attack) is like stabilizing the heart after a myocardial infarction (aka heart attack). Just because the mind is virtual doesn’t mean it is not “real and tangible.” A desktop computer is visible, but the software that brings it to life is invisible. Healing the human mind requires multiple medical interventions by psychiatrists in hospitals and clinics, just like surgeons and endoscopists or cardiologists. Mental health care is as much procedural as other medical and surgical specialties.

One more thing: the validated clinical rating scales for various psychiatric brain disorders (eg, the Positive and Negative Syndrome Scale for schizophrenia, Montgomery-Åsberg Depression Rating Scale for depression, Young Mania Rating Scale for bipolar mania, Hamilton Anxiety Rating Scale for anxiety, Yale-Brown Obsessive Compulsive Scale for obsessive-compulsive disorder) are actual measurement procedures for the severity of the illness, just as a sphygmomanometer measures blood pressure and its improvement with treatment. There are also multiple cognitive test batteries to measure cognitive impairment.7

Finally, unlike psychiatric reimbursement, which is tethered to time, procedures are compensated more generously, irrespective of the time involved. The complexities of diagnosing and treating psychiatric brain disorders that dangerously disrupt thoughts, feelings, behavior, and cognition are just as intricate and demanding as the diagnosis and treatment of general medical and surgical conditions. They should all be equally appreciated as vital life-saving procedures for the human body, brain, and mind.

Many psychiatric physicians lament the dearth of procedures in psychiatry compared to other medical specialties such as surgery, cardiology, gastroenterology, or radiology. The few procedures in psychiatry include electroconvulsive therapy (ECT), repetitive transcranial magnetic stimulation, and vagus nerve stimulation, which are restricted to a small number of sites and not available for most psychiatric practitioners. This lack of tangible/physical procedures should not be surprising because psychiatry deals with disorders of the mind, which are invisible.

However, when one closely examines what psychiatrists do in daily practice to heal our patients, most of what we do actually qualifies as “procedures” although no hardware, machines, or gadgets are involved. Treating psychiatric brain disorders (aka mental illness) requires exquisite skills and expertise, just like medical specialties that use machines to measure or treat various body organs.

It’s time to relabel psychiatric interventions as procedures designed to improve anomalous thoughts, affect, emotions, cognition, and behavior. After giving it some thought (and with a bit of tongue in cheek), I came up with the following list of “psychiatric procedures”:

  • Psychosocial exploratory laparotomy: The comprehensive psychiatric assessment and mental status exam.
  • Chemotherapy: Oral or injective pharmacotherapeutic intervention.
  • Psychoplastic repair: Neuroplasticity, including neurogenesis, synaptogenesis, and dendritic spine regeneration, have been shown to be associated with both psychotherapy and psychotropic medications.1,2
  • Suicidectomy: Extracting the lethal urge to die by suicide.
  • Anger debridement: Removing the irritability and destructive anger outbursts frequently associated with various psychopathologies.
  • Anxiety ablation: Eliminating the noxious emotional state of anxiety and frightening panic attacks.
  • Empathy infusion: Enabling patients to become more understanding of other people and bolstering their impaired “theory of mind.”
  • Personality transplant: Replacing a maladaptive personality with a healthier one (eg, using dialectical behavior therapy for borderline personality disorder).
  • Cognitive LASIK: To improve insight, analogous to how ophthalmologic LASIK improves sight.
  • Mental embolectomy: Removing a blockage to repair rigid attitudes and develop “open-mindedness.”
  • Behavioral dilation and curettage (D&C): To rid patients of negative attributes such as impulsivity or reckless behavior.
  • Psychotherapeutic anesthesia: Numbing emotional pain or severe grief reaction.
  • Social anastomosis: Helping patients who are schizoid or isolative via group therapy, an effective interpersonal and social procedure.
  • Psychotherapeutic stent: To open the vessels of narrow-mindedness.
  • Cortico-psychological resuscitation (CPR): For patients experiencing stress-induced behavioral arrhythmias or emotional infarction.
  • Immunotherapy: Using various neuroprotective psychotropic medications with anti-inflammatory properties or employing evidence-based psychotherapy such as cognitive-behavior therapy (aka neuropsychotherapy), which have been shown to reduce inflammatory biomarkers such as C-reactive protein and cytokines.3
  • Psychotherapy: A neuromodulation procedure for a variety of psychiatric disorders.4
  • Neurobiological facelift: It is well established that neurogenesis, synaptogenesis, and dendritic spine sprouting are significantly increased with both neuroprotective psychotropic medications (antidepressants, lithium, valproate, and second-generationantipsychotics5) as well as with psych­otherapy. There is growing evidence of “premature brain aging” in schizophrenia, bipolar disorder, and depression, with shrinkage in the volume of the cortex and subcortical regions, especially the hippocampus. Psychiatric biopsychosocial interven­tion rebuilds those brain regions by stimulating and replenishing the neuropil and neuro­genic regions (dentate gyrus and subventricular zone). This is like performing virtual plastic surgery on a wrinkled brain and its sagging mind. MRI scans before and after ECT show a remarkable ≥10% increase in the volume of the hippocampus and amygdala, which translates to billions of new neurons, glia, and synapses.6

Reinventing psychiatric therapies as procedures may elicit sarcasm from skeptics, but when you think about it, it is justified. Excising depression is like excising a tumor, not with a scalpel, but virtually. Stabilizing the broken brain and mind after a psychotic episode (aka brain attack) is like stabilizing the heart after a myocardial infarction (aka heart attack). Just because the mind is virtual doesn’t mean it is not “real and tangible.” A desktop computer is visible, but the software that brings it to life is invisible. Healing the human mind requires multiple medical interventions by psychiatrists in hospitals and clinics, just like surgeons and endoscopists or cardiologists. Mental health care is as much procedural as other medical and surgical specialties.

One more thing: the validated clinical rating scales for various psychiatric brain disorders (eg, the Positive and Negative Syndrome Scale for schizophrenia, Montgomery-Åsberg Depression Rating Scale for depression, Young Mania Rating Scale for bipolar mania, Hamilton Anxiety Rating Scale for anxiety, Yale-Brown Obsessive Compulsive Scale for obsessive-compulsive disorder) are actual measurement procedures for the severity of the illness, just as a sphygmomanometer measures blood pressure and its improvement with treatment. There are also multiple cognitive test batteries to measure cognitive impairment.7

Finally, unlike psychiatric reimbursement, which is tethered to time, procedures are compensated more generously, irrespective of the time involved. The complexities of diagnosing and treating psychiatric brain disorders that dangerously disrupt thoughts, feelings, behavior, and cognition are just as intricate and demanding as the diagnosis and treatment of general medical and surgical conditions. They should all be equally appreciated as vital life-saving procedures for the human body, brain, and mind.

References

1. Nasrallah HA, Hopkins T, Pixley SK. Differential effects of antipsychotic and antidepressant drugs on neurogenic regions in rats. Brain Res. 2010;1354:23-29.

2. Tomasino B, Fabbro F. Increases in the right dorsolateral prefrontal cortex and decreases the rostral prefrontal cortex activation after-8 weeks of focused attention based mindfulness meditation. Brain Cogn. 2016;102:46-54.

3. Nasrallah HA. Repositioning psychotherapy as a neurobiological intervention. Current Psychiatry. 2013;12(12):18-19.

4. Nasrallah HA. Optimal psychiatric treatment: Target the brain and avoid the body. Current Psychiatry. 2022;21(12):3-6.

5. Chen AT, Nasrallah HA. Neuroprotective effects of the second generation antipsychotics. Schizophr Res. 2019;208:1-7.

6. Gryglewski G, Lanzenberger R, Silberbauer LR, et al. Meta-analysis of brain structural changes after electroconvulsive therapy in depression. Brain Stimul. 2021;14(4):927-937.

7. Nasrallah HA. The Cognition Self-Assessment Rating Scale for patients with schizophrenia. Current Psychiatry. 2023;22(3):30-34.

References

1. Nasrallah HA, Hopkins T, Pixley SK. Differential effects of antipsychotic and antidepressant drugs on neurogenic regions in rats. Brain Res. 2010;1354:23-29.

2. Tomasino B, Fabbro F. Increases in the right dorsolateral prefrontal cortex and decreases the rostral prefrontal cortex activation after-8 weeks of focused attention based mindfulness meditation. Brain Cogn. 2016;102:46-54.

3. Nasrallah HA. Repositioning psychotherapy as a neurobiological intervention. Current Psychiatry. 2013;12(12):18-19.

4. Nasrallah HA. Optimal psychiatric treatment: Target the brain and avoid the body. Current Psychiatry. 2022;21(12):3-6.

5. Chen AT, Nasrallah HA. Neuroprotective effects of the second generation antipsychotics. Schizophr Res. 2019;208:1-7.

6. Gryglewski G, Lanzenberger R, Silberbauer LR, et al. Meta-analysis of brain structural changes after electroconvulsive therapy in depression. Brain Stimul. 2021;14(4):927-937.

7. Nasrallah HA. The Cognition Self-Assessment Rating Scale for patients with schizophrenia. Current Psychiatry. 2023;22(3):30-34.

Issue
Current Psychiatry - 22(3)
Issue
Current Psychiatry - 22(3)
Page Number
4-5,12
Page Number
4-5,12
Publications
Publications
Article Type
Display Headline
Reimagining psychiatric assessment and interventions as procedures
Display Headline
Reimagining psychiatric assessment and interventions as procedures
Sections
Disallow All Ads
Content Gating
No Gating (article Unlocked/Free)
Alternative CME
Disqus Comments
Default
Use ProPublica
Hide sidebar & use full width
render the right sidebar.
Conference Recap Checkbox
Not Conference Recap
Clinical Edge
Display the Slideshow in this Article
Medscape Article
Display survey writer
Reuters content
Disable Inline Native ads
WebMD Article
Article PDF Media

Evolutions in endoscopy

Article Type
Changed
Wed, 03/01/2023 - 00:15

Dear colleagues,

We continue our theme of highlighting innovations in gastroenterology by exploring how endoscopy continues to blur the lines with surgery. In this issue of Perspectives, Dr. RJ Sealock, assistant professor of medicine at the Baylor College of Medicine, and Dr. Thiru Muniraj, associate professor of medicine at the Yale School of Medicine share their experiences performing minimally invasive alternatives to surgery, discussing both sides of gastrointestinal perforations – treating and creating. Dr. Sealock describes how we can “MacGyver” traditional surgical wound vacs to treat Boerhaave's, while Dr. Muniraj shows how lumen-apposing metal stents allow us to treat acute cholecystitis in poor surgical candidates. 

Dr. Gyanprakash Ketwaroo
We look forward to hearing your thoughts on how endoscopy will continue to evolve @AGA_GIHN.

Gyanprakash A. Ketwaroo, MD, MSc, is associate professor of medicine, Yale University, New Haven, Conn., and chief of endoscopy at West Haven (Conn.) VA Medical Center. He is an associate editor for GI & Hepatology News.
 

Endoscopic vacuum therapy for GI perforation

BY ROBERT JAY SEALOCK, MD

Gastrointestinal endoscopy has evolved from a diagnostic modality into a therapeutic tool used to treat a wide variety of luminal pathology. Endoscopic closure of full thickness injuries is a field that has rapidly expanded because of advanced endoscopic tissue resection and the need for subsequent defect closure as well as technological advances in closure devices such an endoscopic suturing platforms and large over-the-scope clips.

Dr. Robert Jay Sealock

Prior to the advent of closure devices, endoscopic means of treating full thickness defects included through-the-scope (TTS) clips and fully covered metal stents. Given the small size, TTS clips are useful for mucosal closure but are limited in their ability to achieve full thickness closure. Fully covered metal stents utilized particularly for upper GI tract perforations and leaks are intended to divert gastrointestinal content away from the site of injury, thereby allowing secondary intention healing. Stents have several limitations, including frequent downstream migration and an inability to create a “watertight” seal in minimizing wound contamination. For decades, our surgical colleagues have utilized negative pressure wound therapy or vacuum therapy to expedite large wound closure. Given their familiarity with the technique, surgeons began adapting vacuum therapy for the treatment of postsurgical anastomotic leaks and fistulas particularly within the rectum.1 Eventually, the same technique was applied to the treatment of upper GI tract anastomotic leaks.2 Endoscopic vacuum therapy (EVT) overcomes many of the limitations of traditional endoscopic closure or diversion using covered stents through the use of suction to promote granulation tissue and aspirate infected wound contents.3

The approach to full thickness luminal injury must be individualized, but for a majority of indications EVT can be considered as a first-line approach. In our own experience, EVT closure can be achieved in more than 80% of patients with a variety of injuries such as iatrogenic endoscopic perforations (e.g., esophageal perforation during Savary dilation), surgical defects (sleeve gastrectomy leaks), and spontaneous perforations (e.g., Boerhaave syndrome). The initial step is endoscopic assessment of the luminal injury as well as the extraluminal cavity. In some situations, it is necessary to manually clean the defect cavity of necrotic material and food.

Once the cavity is cleaned and the size of the defect is assessed, the EVT device is manufactured at the bedside using commonly available materials and tools. A wound vacuum polyurethane sponge is affixed to a nasogastric tube, trimmed to the desired shape and size, and placed either within the defect cavity or within the GI lumen next to the defect opening.4 The EVT device is exchanged at an interval of 3-5 days, which allows the promotion of granulation tissue and subsequent downsizing as the cavity shrinks. In our series, an average number of five exchanges was necessary to achieve closure, with an average time to closure of 25 days.

Most experts would recommend initially placing the EVT device within the defect cavity. Once the cavity size can no longer accommodate the device, complete closure is achieved via intraluminal placement. The use of constant negative pressure (typically 150 mm to 175 mm Hg) prevents migration or dislodgement of the device.

For those who use EVT, there is some satisfaction from assembling and tailoring your own device, much like the protagonist in the 1980s television series “MacGyver,” who would manufacture devices out of readily available materials to address difficult and life-threatening situations. This need for self-assembly also has fostered ingenuity and creativity in the field, which can be found in social media and peer-reviewed sources.5 For some, however, the need to assemble your own device may be a deterrent. There is certainly an opportunity for commercialization and innovation, thereby putting Food and Drug Administration–approved devices into the hands of endoscopists. EVT is also a time- and labor-intensive therapy without specific reimbursement codes. Despite these limitations we continue to use and advocate for EVT given its clinical success in a population of patients with complex luminal injuries.


Dr. Sealock is assistant professor of medicine, department of gastroenterology and hepatology, Baylor College of Medicine, Houston. He receives research funding from AbbVie and is a consultant to ConMed and Ambu.
 

References

1. Weidenhagen R et al. Endoscopic vacuum-assisted closure of anastomotic leakage following anterior resection of the rectum: A new method. Surg Endosc Other Interv Tech. 2008;22(8):1818-25. doi: 10.1007/s00464-007-9706-x.

2. Wedemeyer J et al. Endoscopic vacuum-assisted closure of upper intestinal anastomotic leaks. Gastrointest Endosc. 2008;67(4):708-11. doi: 10.1016/j.gie.2007.10.064.

3. Mennigen R et al. Comparison of endoscopic vacuum therapy versus stent for anastomotic leak after esophagectomy. J Gastrointest Surg. 2015;19(7):1229-35.

4. Abdulsada M et al. Endoluminal vacuum therapy of esophageal perforations. VideoGIE. 2020;5(1):8-10. doi: 10.1016/j.vgie.2019.10.004

5. de Moura DTH et al. Cost-effective modified endoscopic vacuum therapy for the treatment of gastrointestinal transmural defects: Step-by-step process of manufacturing and its advantages. VideoGIE. 2021 Sep 4;6(12):523-8. doi: 10.1016/j.vgie.2021.08.002.

 

 

LAMS for gallbladder drainage

BY THIRU MUNIRAJ, MD, PHD, FACG, FRCP

Surgical cholecystectomy is the gold standard of treatment for acute cholecystitis (AC).1 The morbidity and mortality rates remain high in high-risk surgical patients, such as those with cirrhosis, coagulopathy, advanced malignancy, severe cardiopulmonary conditions, or poor performance status. Percutaneous gallbladder drainage (PT-GBD) typically has been performed as an alternative in these cases. Endoscopic ultrasound-guided gallbladder drainage (EUS-GBD) is rapidly becoming a preferred alternative treatment to surgery in the case of AC at expert centers.

Dr. Thiru Muniraj

Since Baron and Topazian introduced EUS-GBD using a double pigtail stent in 2007, the procedure has evolved with the introduction of dedicated newly developed short, bi-flanged, covered lumen-apposing metal stents (LAMS) that have revolutionized this procedure as a single-step technique with excellent efficacy and safety outcomes. Although EUS-GBD is widely adopted among endosonographers, several skilled ERCP [endoscopic retrograde cholangiopancreatography] endoscopists still perform endoscopic transpapillary gallbladder drainage (ET-GBD) with ERCP as an alternative for high-risk surgical patients with AC. However, three-way comparative studies and randomized trials between PT-GBD, ETGBD, and EUS-GBD have clearly shown that EUS-GBD with LAMS is the most effective and safer alternative with the lowest rate of recurrent cholecystitis.2,3 The recent Tokyo Guidelines 2018 now suggest EUS-GBD as one of the viable options for AC treatment.4

In my institution, we offer EUS-GBD for nonsurgical candidates with AC with and without gallstones. In addition to its excellent benefits on quality of life through avoidance of an external percutaneous drain, EUS-GBD offers the ability to remove gallstones endoscopically using irrigation, suction, basket, and direct electrohydraulic lithotripsy. Moreover, EUS-GBD allows direct visualization and mucosal evaluation of the gallbladder when dysplasia or malignancy is suspected. The other indications where I perform EUS-GBD drainage are conversion of PT-GBD to EUS-GBD and as a backdoor alternate to failed ERCP where the cystic duct is patent and EUS-bile duct drainage is not amenable. In nonoperative malignant biliary stricture patients with indwelling metal biliary stents covering the cystic duct, I have a low threshold to perform a prophylactic EUS-GBD if the gallbladder is distended.

I perform EUS-GBD procedures under propofol intravenous anesthesia with the patient in the left lateral position on the fluoroscopy table. I choose the site to create the fistula for EUS-GBD either in the duodenal bulb or gastric antrum, whichever seems safer and easier to deploy the LAMS stent without torquing the endoscope much. In case of inadvertent complications such as stent maldeployment, the gastric site is often very forgiving. My preferred stent for EUS-GBD is 10 mm x 10 mm LAMS with hot cautery, as this seems to be the ideal size. We can choose a 10 mm x 15 mm stent if a larger stone removal is expected. I never choose smaller LAMS stents (6 mm and 8 mm), as the saddle length is not enough to bridge the thickened gallbladder wall and the thick gastric antral wall. In patients with calculous cholecystitis, I prefer to place a 7Fr 4cm pigtail plastic stent within the lumen of LAMS to ensure patency, especially if it is a gastric site, as food occlusion is more common. Unlike with pseudocyst drainage, these LAMS for EUS-GBD can be left indefinitely without removal. I avoid EUS-GBD in patients who have large-volume ascites or are too sick to tolerate anesthesia. Although a subsequent cholecystectomy post EUS-GBD is doable, I have a clear discussion with the surgeon before choosing this approach over ERCP ET-GBD in case future surgery is still an option. This is more important in patients who are awaiting liver transplantation.

The first step in establishing a program for EUS-GBD is to establish strong collaboration with your surgeons. In our institution, once our surgeons determine that patients with AC are high risk for surgery, they initiate a multidisciplinary discussion and reach out to advanced endoscopists at the same time or before consulting interventional radiology. The key to establishing a successful EUS-GBD program is to get “buy-in” from the surgeons and create a “signature” pathway for AC in your own institution.

EUS-GBD to drain the gallbladder in nonsurgical patients is one of my favorite procedures. Until the currently available LAMS secures an on-label indication for AC, we must wait and watch to see if there are enough advanced endoscopists ready to take over the challenge of all nonsurgical cholecystitis gallbladders – especially during late-night calls – rather than requesting PT-GBD. Soon, EUS-GBD will consign PT-GBD to centers without access to advanced endoscopists who perform EUS-guided interventions and limit ERCP transpapillary ET-GBD to patients with coagulopathy or large ascites.

Dr. Muniraj is associate professor of medicine, Yale School of Medicine, New Haven, Conn., and a consultant to Boston Scientific.
 

References

1. Endo I et al. Optimal treatment strategy for acute cholecystitis based on predictive factors: Japan-Taiwan multicenter cohort study. J Hepatobiliary Pancreat Sci. 2017. 24(6):346-61.

2. Siddiqui A et al. Three-way comparative study of endoscopic ultrasound-guided transmural gallbladder drainage using lumen-apposing metal stents versus endoscopic transpapillary drainage versus percutaneous cholecystostomy for gallbladder drainage in high-risk surgical patients with acute cholecystitis: clinical outcomes and success in an international, multicenter study. Surg Endosc. 2019;33(4):1260-70.

3. Teoh AYB et al. Endosonography-guided gallbladder drainage versus percutaneous cholecystostomy in very high-risk surgical patients with acute cholecystitis: An international randomised multicentre controlled superiority trial (DRAC 1). Gut. 2020;69(6):1085-91.

4. Mori Y et al. Tokyo Guidelines 2018: Management strategies for gallbladder drainage in patients with acute cholecystitis (with videos). J Hepatobiliary Pancreat Sci. 2018;25(1):87-95.

Publications
Topics
Sections

Dear colleagues,

We continue our theme of highlighting innovations in gastroenterology by exploring how endoscopy continues to blur the lines with surgery. In this issue of Perspectives, Dr. RJ Sealock, assistant professor of medicine at the Baylor College of Medicine, and Dr. Thiru Muniraj, associate professor of medicine at the Yale School of Medicine share their experiences performing minimally invasive alternatives to surgery, discussing both sides of gastrointestinal perforations – treating and creating. Dr. Sealock describes how we can “MacGyver” traditional surgical wound vacs to treat Boerhaave's, while Dr. Muniraj shows how lumen-apposing metal stents allow us to treat acute cholecystitis in poor surgical candidates. 

Dr. Gyanprakash Ketwaroo
We look forward to hearing your thoughts on how endoscopy will continue to evolve @AGA_GIHN.

Gyanprakash A. Ketwaroo, MD, MSc, is associate professor of medicine, Yale University, New Haven, Conn., and chief of endoscopy at West Haven (Conn.) VA Medical Center. He is an associate editor for GI & Hepatology News.
 

Endoscopic vacuum therapy for GI perforation

BY ROBERT JAY SEALOCK, MD

Gastrointestinal endoscopy has evolved from a diagnostic modality into a therapeutic tool used to treat a wide variety of luminal pathology. Endoscopic closure of full thickness injuries is a field that has rapidly expanded because of advanced endoscopic tissue resection and the need for subsequent defect closure as well as technological advances in closure devices such an endoscopic suturing platforms and large over-the-scope clips.

Dr. Robert Jay Sealock

Prior to the advent of closure devices, endoscopic means of treating full thickness defects included through-the-scope (TTS) clips and fully covered metal stents. Given the small size, TTS clips are useful for mucosal closure but are limited in their ability to achieve full thickness closure. Fully covered metal stents utilized particularly for upper GI tract perforations and leaks are intended to divert gastrointestinal content away from the site of injury, thereby allowing secondary intention healing. Stents have several limitations, including frequent downstream migration and an inability to create a “watertight” seal in minimizing wound contamination. For decades, our surgical colleagues have utilized negative pressure wound therapy or vacuum therapy to expedite large wound closure. Given their familiarity with the technique, surgeons began adapting vacuum therapy for the treatment of postsurgical anastomotic leaks and fistulas particularly within the rectum.1 Eventually, the same technique was applied to the treatment of upper GI tract anastomotic leaks.2 Endoscopic vacuum therapy (EVT) overcomes many of the limitations of traditional endoscopic closure or diversion using covered stents through the use of suction to promote granulation tissue and aspirate infected wound contents.3

The approach to full thickness luminal injury must be individualized, but for a majority of indications EVT can be considered as a first-line approach. In our own experience, EVT closure can be achieved in more than 80% of patients with a variety of injuries such as iatrogenic endoscopic perforations (e.g., esophageal perforation during Savary dilation), surgical defects (sleeve gastrectomy leaks), and spontaneous perforations (e.g., Boerhaave syndrome). The initial step is endoscopic assessment of the luminal injury as well as the extraluminal cavity. In some situations, it is necessary to manually clean the defect cavity of necrotic material and food.

Once the cavity is cleaned and the size of the defect is assessed, the EVT device is manufactured at the bedside using commonly available materials and tools. A wound vacuum polyurethane sponge is affixed to a nasogastric tube, trimmed to the desired shape and size, and placed either within the defect cavity or within the GI lumen next to the defect opening.4 The EVT device is exchanged at an interval of 3-5 days, which allows the promotion of granulation tissue and subsequent downsizing as the cavity shrinks. In our series, an average number of five exchanges was necessary to achieve closure, with an average time to closure of 25 days.

Most experts would recommend initially placing the EVT device within the defect cavity. Once the cavity size can no longer accommodate the device, complete closure is achieved via intraluminal placement. The use of constant negative pressure (typically 150 mm to 175 mm Hg) prevents migration or dislodgement of the device.

For those who use EVT, there is some satisfaction from assembling and tailoring your own device, much like the protagonist in the 1980s television series “MacGyver,” who would manufacture devices out of readily available materials to address difficult and life-threatening situations. This need for self-assembly also has fostered ingenuity and creativity in the field, which can be found in social media and peer-reviewed sources.5 For some, however, the need to assemble your own device may be a deterrent. There is certainly an opportunity for commercialization and innovation, thereby putting Food and Drug Administration–approved devices into the hands of endoscopists. EVT is also a time- and labor-intensive therapy without specific reimbursement codes. Despite these limitations we continue to use and advocate for EVT given its clinical success in a population of patients with complex luminal injuries.


Dr. Sealock is assistant professor of medicine, department of gastroenterology and hepatology, Baylor College of Medicine, Houston. He receives research funding from AbbVie and is a consultant to ConMed and Ambu.
 

References

1. Weidenhagen R et al. Endoscopic vacuum-assisted closure of anastomotic leakage following anterior resection of the rectum: A new method. Surg Endosc Other Interv Tech. 2008;22(8):1818-25. doi: 10.1007/s00464-007-9706-x.

2. Wedemeyer J et al. Endoscopic vacuum-assisted closure of upper intestinal anastomotic leaks. Gastrointest Endosc. 2008;67(4):708-11. doi: 10.1016/j.gie.2007.10.064.

3. Mennigen R et al. Comparison of endoscopic vacuum therapy versus stent for anastomotic leak after esophagectomy. J Gastrointest Surg. 2015;19(7):1229-35.

4. Abdulsada M et al. Endoluminal vacuum therapy of esophageal perforations. VideoGIE. 2020;5(1):8-10. doi: 10.1016/j.vgie.2019.10.004

5. de Moura DTH et al. Cost-effective modified endoscopic vacuum therapy for the treatment of gastrointestinal transmural defects: Step-by-step process of manufacturing and its advantages. VideoGIE. 2021 Sep 4;6(12):523-8. doi: 10.1016/j.vgie.2021.08.002.

 

 

LAMS for gallbladder drainage

BY THIRU MUNIRAJ, MD, PHD, FACG, FRCP

Surgical cholecystectomy is the gold standard of treatment for acute cholecystitis (AC).1 The morbidity and mortality rates remain high in high-risk surgical patients, such as those with cirrhosis, coagulopathy, advanced malignancy, severe cardiopulmonary conditions, or poor performance status. Percutaneous gallbladder drainage (PT-GBD) typically has been performed as an alternative in these cases. Endoscopic ultrasound-guided gallbladder drainage (EUS-GBD) is rapidly becoming a preferred alternative treatment to surgery in the case of AC at expert centers.

Dr. Thiru Muniraj

Since Baron and Topazian introduced EUS-GBD using a double pigtail stent in 2007, the procedure has evolved with the introduction of dedicated newly developed short, bi-flanged, covered lumen-apposing metal stents (LAMS) that have revolutionized this procedure as a single-step technique with excellent efficacy and safety outcomes. Although EUS-GBD is widely adopted among endosonographers, several skilled ERCP [endoscopic retrograde cholangiopancreatography] endoscopists still perform endoscopic transpapillary gallbladder drainage (ET-GBD) with ERCP as an alternative for high-risk surgical patients with AC. However, three-way comparative studies and randomized trials between PT-GBD, ETGBD, and EUS-GBD have clearly shown that EUS-GBD with LAMS is the most effective and safer alternative with the lowest rate of recurrent cholecystitis.2,3 The recent Tokyo Guidelines 2018 now suggest EUS-GBD as one of the viable options for AC treatment.4

In my institution, we offer EUS-GBD for nonsurgical candidates with AC with and without gallstones. In addition to its excellent benefits on quality of life through avoidance of an external percutaneous drain, EUS-GBD offers the ability to remove gallstones endoscopically using irrigation, suction, basket, and direct electrohydraulic lithotripsy. Moreover, EUS-GBD allows direct visualization and mucosal evaluation of the gallbladder when dysplasia or malignancy is suspected. The other indications where I perform EUS-GBD drainage are conversion of PT-GBD to EUS-GBD and as a backdoor alternate to failed ERCP where the cystic duct is patent and EUS-bile duct drainage is not amenable. In nonoperative malignant biliary stricture patients with indwelling metal biliary stents covering the cystic duct, I have a low threshold to perform a prophylactic EUS-GBD if the gallbladder is distended.

I perform EUS-GBD procedures under propofol intravenous anesthesia with the patient in the left lateral position on the fluoroscopy table. I choose the site to create the fistula for EUS-GBD either in the duodenal bulb or gastric antrum, whichever seems safer and easier to deploy the LAMS stent without torquing the endoscope much. In case of inadvertent complications such as stent maldeployment, the gastric site is often very forgiving. My preferred stent for EUS-GBD is 10 mm x 10 mm LAMS with hot cautery, as this seems to be the ideal size. We can choose a 10 mm x 15 mm stent if a larger stone removal is expected. I never choose smaller LAMS stents (6 mm and 8 mm), as the saddle length is not enough to bridge the thickened gallbladder wall and the thick gastric antral wall. In patients with calculous cholecystitis, I prefer to place a 7Fr 4cm pigtail plastic stent within the lumen of LAMS to ensure patency, especially if it is a gastric site, as food occlusion is more common. Unlike with pseudocyst drainage, these LAMS for EUS-GBD can be left indefinitely without removal. I avoid EUS-GBD in patients who have large-volume ascites or are too sick to tolerate anesthesia. Although a subsequent cholecystectomy post EUS-GBD is doable, I have a clear discussion with the surgeon before choosing this approach over ERCP ET-GBD in case future surgery is still an option. This is more important in patients who are awaiting liver transplantation.

The first step in establishing a program for EUS-GBD is to establish strong collaboration with your surgeons. In our institution, once our surgeons determine that patients with AC are high risk for surgery, they initiate a multidisciplinary discussion and reach out to advanced endoscopists at the same time or before consulting interventional radiology. The key to establishing a successful EUS-GBD program is to get “buy-in” from the surgeons and create a “signature” pathway for AC in your own institution.

EUS-GBD to drain the gallbladder in nonsurgical patients is one of my favorite procedures. Until the currently available LAMS secures an on-label indication for AC, we must wait and watch to see if there are enough advanced endoscopists ready to take over the challenge of all nonsurgical cholecystitis gallbladders – especially during late-night calls – rather than requesting PT-GBD. Soon, EUS-GBD will consign PT-GBD to centers without access to advanced endoscopists who perform EUS-guided interventions and limit ERCP transpapillary ET-GBD to patients with coagulopathy or large ascites.

Dr. Muniraj is associate professor of medicine, Yale School of Medicine, New Haven, Conn., and a consultant to Boston Scientific.
 

References

1. Endo I et al. Optimal treatment strategy for acute cholecystitis based on predictive factors: Japan-Taiwan multicenter cohort study. J Hepatobiliary Pancreat Sci. 2017. 24(6):346-61.

2. Siddiqui A et al. Three-way comparative study of endoscopic ultrasound-guided transmural gallbladder drainage using lumen-apposing metal stents versus endoscopic transpapillary drainage versus percutaneous cholecystostomy for gallbladder drainage in high-risk surgical patients with acute cholecystitis: clinical outcomes and success in an international, multicenter study. Surg Endosc. 2019;33(4):1260-70.

3. Teoh AYB et al. Endosonography-guided gallbladder drainage versus percutaneous cholecystostomy in very high-risk surgical patients with acute cholecystitis: An international randomised multicentre controlled superiority trial (DRAC 1). Gut. 2020;69(6):1085-91.

4. Mori Y et al. Tokyo Guidelines 2018: Management strategies for gallbladder drainage in patients with acute cholecystitis (with videos). J Hepatobiliary Pancreat Sci. 2018;25(1):87-95.

Dear colleagues,

We continue our theme of highlighting innovations in gastroenterology by exploring how endoscopy continues to blur the lines with surgery. In this issue of Perspectives, Dr. RJ Sealock, assistant professor of medicine at the Baylor College of Medicine, and Dr. Thiru Muniraj, associate professor of medicine at the Yale School of Medicine share their experiences performing minimally invasive alternatives to surgery, discussing both sides of gastrointestinal perforations – treating and creating. Dr. Sealock describes how we can “MacGyver” traditional surgical wound vacs to treat Boerhaave's, while Dr. Muniraj shows how lumen-apposing metal stents allow us to treat acute cholecystitis in poor surgical candidates. 

Dr. Gyanprakash Ketwaroo
We look forward to hearing your thoughts on how endoscopy will continue to evolve @AGA_GIHN.

Gyanprakash A. Ketwaroo, MD, MSc, is associate professor of medicine, Yale University, New Haven, Conn., and chief of endoscopy at West Haven (Conn.) VA Medical Center. He is an associate editor for GI & Hepatology News.
 

Endoscopic vacuum therapy for GI perforation

BY ROBERT JAY SEALOCK, MD

Gastrointestinal endoscopy has evolved from a diagnostic modality into a therapeutic tool used to treat a wide variety of luminal pathology. Endoscopic closure of full thickness injuries is a field that has rapidly expanded because of advanced endoscopic tissue resection and the need for subsequent defect closure as well as technological advances in closure devices such an endoscopic suturing platforms and large over-the-scope clips.

Dr. Robert Jay Sealock

Prior to the advent of closure devices, endoscopic means of treating full thickness defects included through-the-scope (TTS) clips and fully covered metal stents. Given the small size, TTS clips are useful for mucosal closure but are limited in their ability to achieve full thickness closure. Fully covered metal stents utilized particularly for upper GI tract perforations and leaks are intended to divert gastrointestinal content away from the site of injury, thereby allowing secondary intention healing. Stents have several limitations, including frequent downstream migration and an inability to create a “watertight” seal in minimizing wound contamination. For decades, our surgical colleagues have utilized negative pressure wound therapy or vacuum therapy to expedite large wound closure. Given their familiarity with the technique, surgeons began adapting vacuum therapy for the treatment of postsurgical anastomotic leaks and fistulas particularly within the rectum.1 Eventually, the same technique was applied to the treatment of upper GI tract anastomotic leaks.2 Endoscopic vacuum therapy (EVT) overcomes many of the limitations of traditional endoscopic closure or diversion using covered stents through the use of suction to promote granulation tissue and aspirate infected wound contents.3

The approach to full thickness luminal injury must be individualized, but for a majority of indications EVT can be considered as a first-line approach. In our own experience, EVT closure can be achieved in more than 80% of patients with a variety of injuries such as iatrogenic endoscopic perforations (e.g., esophageal perforation during Savary dilation), surgical defects (sleeve gastrectomy leaks), and spontaneous perforations (e.g., Boerhaave syndrome). The initial step is endoscopic assessment of the luminal injury as well as the extraluminal cavity. In some situations, it is necessary to manually clean the defect cavity of necrotic material and food.

Once the cavity is cleaned and the size of the defect is assessed, the EVT device is manufactured at the bedside using commonly available materials and tools. A wound vacuum polyurethane sponge is affixed to a nasogastric tube, trimmed to the desired shape and size, and placed either within the defect cavity or within the GI lumen next to the defect opening.4 The EVT device is exchanged at an interval of 3-5 days, which allows the promotion of granulation tissue and subsequent downsizing as the cavity shrinks. In our series, an average number of five exchanges was necessary to achieve closure, with an average time to closure of 25 days.

Most experts would recommend initially placing the EVT device within the defect cavity. Once the cavity size can no longer accommodate the device, complete closure is achieved via intraluminal placement. The use of constant negative pressure (typically 150 mm to 175 mm Hg) prevents migration or dislodgement of the device.

For those who use EVT, there is some satisfaction from assembling and tailoring your own device, much like the protagonist in the 1980s television series “MacGyver,” who would manufacture devices out of readily available materials to address difficult and life-threatening situations. This need for self-assembly also has fostered ingenuity and creativity in the field, which can be found in social media and peer-reviewed sources.5 For some, however, the need to assemble your own device may be a deterrent. There is certainly an opportunity for commercialization and innovation, thereby putting Food and Drug Administration–approved devices into the hands of endoscopists. EVT is also a time- and labor-intensive therapy without specific reimbursement codes. Despite these limitations we continue to use and advocate for EVT given its clinical success in a population of patients with complex luminal injuries.


Dr. Sealock is assistant professor of medicine, department of gastroenterology and hepatology, Baylor College of Medicine, Houston. He receives research funding from AbbVie and is a consultant to ConMed and Ambu.
 

References

1. Weidenhagen R et al. Endoscopic vacuum-assisted closure of anastomotic leakage following anterior resection of the rectum: A new method. Surg Endosc Other Interv Tech. 2008;22(8):1818-25. doi: 10.1007/s00464-007-9706-x.

2. Wedemeyer J et al. Endoscopic vacuum-assisted closure of upper intestinal anastomotic leaks. Gastrointest Endosc. 2008;67(4):708-11. doi: 10.1016/j.gie.2007.10.064.

3. Mennigen R et al. Comparison of endoscopic vacuum therapy versus stent for anastomotic leak after esophagectomy. J Gastrointest Surg. 2015;19(7):1229-35.

4. Abdulsada M et al. Endoluminal vacuum therapy of esophageal perforations. VideoGIE. 2020;5(1):8-10. doi: 10.1016/j.vgie.2019.10.004

5. de Moura DTH et al. Cost-effective modified endoscopic vacuum therapy for the treatment of gastrointestinal transmural defects: Step-by-step process of manufacturing and its advantages. VideoGIE. 2021 Sep 4;6(12):523-8. doi: 10.1016/j.vgie.2021.08.002.

 

 

LAMS for gallbladder drainage

BY THIRU MUNIRAJ, MD, PHD, FACG, FRCP

Surgical cholecystectomy is the gold standard of treatment for acute cholecystitis (AC).1 The morbidity and mortality rates remain high in high-risk surgical patients, such as those with cirrhosis, coagulopathy, advanced malignancy, severe cardiopulmonary conditions, or poor performance status. Percutaneous gallbladder drainage (PT-GBD) typically has been performed as an alternative in these cases. Endoscopic ultrasound-guided gallbladder drainage (EUS-GBD) is rapidly becoming a preferred alternative treatment to surgery in the case of AC at expert centers.

Dr. Thiru Muniraj

Since Baron and Topazian introduced EUS-GBD using a double pigtail stent in 2007, the procedure has evolved with the introduction of dedicated newly developed short, bi-flanged, covered lumen-apposing metal stents (LAMS) that have revolutionized this procedure as a single-step technique with excellent efficacy and safety outcomes. Although EUS-GBD is widely adopted among endosonographers, several skilled ERCP [endoscopic retrograde cholangiopancreatography] endoscopists still perform endoscopic transpapillary gallbladder drainage (ET-GBD) with ERCP as an alternative for high-risk surgical patients with AC. However, three-way comparative studies and randomized trials between PT-GBD, ETGBD, and EUS-GBD have clearly shown that EUS-GBD with LAMS is the most effective and safer alternative with the lowest rate of recurrent cholecystitis.2,3 The recent Tokyo Guidelines 2018 now suggest EUS-GBD as one of the viable options for AC treatment.4

In my institution, we offer EUS-GBD for nonsurgical candidates with AC with and without gallstones. In addition to its excellent benefits on quality of life through avoidance of an external percutaneous drain, EUS-GBD offers the ability to remove gallstones endoscopically using irrigation, suction, basket, and direct electrohydraulic lithotripsy. Moreover, EUS-GBD allows direct visualization and mucosal evaluation of the gallbladder when dysplasia or malignancy is suspected. The other indications where I perform EUS-GBD drainage are conversion of PT-GBD to EUS-GBD and as a backdoor alternate to failed ERCP where the cystic duct is patent and EUS-bile duct drainage is not amenable. In nonoperative malignant biliary stricture patients with indwelling metal biliary stents covering the cystic duct, I have a low threshold to perform a prophylactic EUS-GBD if the gallbladder is distended.

I perform EUS-GBD procedures under propofol intravenous anesthesia with the patient in the left lateral position on the fluoroscopy table. I choose the site to create the fistula for EUS-GBD either in the duodenal bulb or gastric antrum, whichever seems safer and easier to deploy the LAMS stent without torquing the endoscope much. In case of inadvertent complications such as stent maldeployment, the gastric site is often very forgiving. My preferred stent for EUS-GBD is 10 mm x 10 mm LAMS with hot cautery, as this seems to be the ideal size. We can choose a 10 mm x 15 mm stent if a larger stone removal is expected. I never choose smaller LAMS stents (6 mm and 8 mm), as the saddle length is not enough to bridge the thickened gallbladder wall and the thick gastric antral wall. In patients with calculous cholecystitis, I prefer to place a 7Fr 4cm pigtail plastic stent within the lumen of LAMS to ensure patency, especially if it is a gastric site, as food occlusion is more common. Unlike with pseudocyst drainage, these LAMS for EUS-GBD can be left indefinitely without removal. I avoid EUS-GBD in patients who have large-volume ascites or are too sick to tolerate anesthesia. Although a subsequent cholecystectomy post EUS-GBD is doable, I have a clear discussion with the surgeon before choosing this approach over ERCP ET-GBD in case future surgery is still an option. This is more important in patients who are awaiting liver transplantation.

The first step in establishing a program for EUS-GBD is to establish strong collaboration with your surgeons. In our institution, once our surgeons determine that patients with AC are high risk for surgery, they initiate a multidisciplinary discussion and reach out to advanced endoscopists at the same time or before consulting interventional radiology. The key to establishing a successful EUS-GBD program is to get “buy-in” from the surgeons and create a “signature” pathway for AC in your own institution.

EUS-GBD to drain the gallbladder in nonsurgical patients is one of my favorite procedures. Until the currently available LAMS secures an on-label indication for AC, we must wait and watch to see if there are enough advanced endoscopists ready to take over the challenge of all nonsurgical cholecystitis gallbladders – especially during late-night calls – rather than requesting PT-GBD. Soon, EUS-GBD will consign PT-GBD to centers without access to advanced endoscopists who perform EUS-guided interventions and limit ERCP transpapillary ET-GBD to patients with coagulopathy or large ascites.

Dr. Muniraj is associate professor of medicine, Yale School of Medicine, New Haven, Conn., and a consultant to Boston Scientific.
 

References

1. Endo I et al. Optimal treatment strategy for acute cholecystitis based on predictive factors: Japan-Taiwan multicenter cohort study. J Hepatobiliary Pancreat Sci. 2017. 24(6):346-61.

2. Siddiqui A et al. Three-way comparative study of endoscopic ultrasound-guided transmural gallbladder drainage using lumen-apposing metal stents versus endoscopic transpapillary drainage versus percutaneous cholecystostomy for gallbladder drainage in high-risk surgical patients with acute cholecystitis: clinical outcomes and success in an international, multicenter study. Surg Endosc. 2019;33(4):1260-70.

3. Teoh AYB et al. Endosonography-guided gallbladder drainage versus percutaneous cholecystostomy in very high-risk surgical patients with acute cholecystitis: An international randomised multicentre controlled superiority trial (DRAC 1). Gut. 2020;69(6):1085-91.

4. Mori Y et al. Tokyo Guidelines 2018: Management strategies for gallbladder drainage in patients with acute cholecystitis (with videos). J Hepatobiliary Pancreat Sci. 2018;25(1):87-95.

Publications
Publications
Topics
Article Type
Sections
Disallow All Ads
Content Gating
No Gating (article Unlocked/Free)
Alternative CME
Disqus Comments
Default
Use ProPublica
Hide sidebar & use full width
render the right sidebar.
Conference Recap Checkbox
Not Conference Recap
Clinical Edge
Display the Slideshow in this Article
Medscape Article
Display survey writer
Reuters content
Disable Inline Native ads
WebMD Article

Prepare for endometriosis excision surgery with a multidisciplinary approach

Article Type
Changed
Mon, 03/06/2023 - 09:30

Introduction: The preoperative evaluation for endometriosis – more than meets the eye

It is well known that it often takes 6-10 years for endometriosis to be diagnosed in patients who have the disease, depending on where the patient lives. I certainly am not surprised. During my residency at Parkland Memorial Hospital, if a patient had chronic pelvic pain and no fibroids, her diagnosis was usually pelvic inflammatory disease. Later, during my fellowship in reproductive endocrinology at the University of Pennsylvania, the diagnosis became endometriosis.

As I gained more interest and expertise in the treatment of endometriosis, I became aware of several articles concluding that if a woman sought treatment for chronic pelvic pain with an internist, the diagnosis would be irritable bowel syndrome (IBS); with a urologist, it would be interstitial cystitis; and with a gynecologist, endometriosis. Moreover, there is an increased propensity for IBS and IC in patients with endometriosis. There also is an increased risk of small intestine bacterial overgrowth (SIBO), as noted by our guest author for this latest installment of the Master Class in Gynecologic Surgery, Iris Orbuch, MD.

Like our guest author, I have also noted increased risk of pelvic floor myalgia. Dr. Orbuch clearly outlines why this occurs. In fact, we can now understand why many patients have multiple pelvic pain–inducing issues compounding their pain secondary to endometriosis and leading to remodeling of the central nervous system. Therefore, it certainly makes sense to follow Dr. Orbuch’s recommendation for a multidisciplinary pre- and postsurgical approach “to downregulate the pain generators.”

Dr. Orbuch is a minimally invasive gynecologic surgeon in Los Angeles who specializes in the treatment of patients diagnosed with endometriosis. Dr. Orbuch serves on the Board of Directors of the Foundation of the American Association of Gynecologic Laparoscopists and has served as the chair of the AAGL’s Special Interest Group on Endometriosis and Reproductive Surgery. She is the coauthor of the book “Beating Endo – How to Reclaim Your Life From Endometriosis” (New York: HarperCollins; 2019). The book is written for patients but addresses many issues discussed in this installment of the Master Class in Gynecologic Surgery.

Dr. Miller, MD, FACOG, is professor of obstetrics and gynecology, department of clinical sciences, Rosalind Franklin University of Medicine and Science, North Chicago. He has no conflicts of interest to report.

 

 

Patients with endometriosis and the all-too-often decade-long diagnostic delay have a variety of coexisting conditions that are pain generators – from painful bladder syndrome and pelvic floor dysfunction to a small intestine bacterial system that is significantly upregulated and sensitized.

For optimal surgical outcomes, and to help our patients recover from years of this inflammatory, systemic disease, we must treat our patients holistically and work to downregulate their pain as much as possible before excision surgery. I work with patients a few months prior to surgery, often for 4-5 months, during which time they not only see me for informative follow-ups, but also pelvic floor physical therapists, gastroenterologists, mental health professionals, integrative nutritionists, and physiatrists or pain specialists, depending on their needs.1

By identifying coexisting conditions in an initial consult and employing a presurgical multidisciplinary approach to downregulate the pain generators, my patients recover well from excision surgery, with greater and faster relief from pain, compared with those using standard approaches, and with little to no use of opioids.

At a minimum, given the unfortunate time constraints and productivity demands of working within health systems – and considering that surgeries are often scheduled a couple of months out – the surgeon could ensure that patients are engaged in at least 6-8 weeks of pelvic floor physical therapy before surgery to sufficiently lengthen the pelvic muscles and loosen surrounding fascia.

Short, tight pelvic floor muscles are almost universal in patients with delayed diagnosis of endometriosis and are significant generators of pain.
 

Appreciating sequelae of diagnostic delay

After my fellowship in advanced laparoscopic and pelvic surgery with Harry Reich, MD, and C. Y. Liu, MD, pioneers of endometriosis excision surgery, and as I did my residency in the early 2000s, I noticed puzzlement in the literature about why some patients still had lasting pain after thorough excision.

I didn’t doubt the efficacy of excision. It is the cornerstone of treatment, and at least one randomized double-blind trial2 and a systematic review and meta-analysis3 have demonstrated its superior efficacy over ablation in symptom reduction. What I did doubt was any presumption that surgery alone was enough. I knew there was more to healing when a disease process wreaks havoc on the body for more than a decade and that there were other generators of pain in addition to the endometriosis implants themselves.

As I began to focus on endometriosis in my own surgical practice, I strove to detect and treat endometriosis in teens. But in those patients with longstanding disease, I recognized patterns and began to more fully appreciate the systemic sequelae of endometriosis.

To cope with dysmenorrhea, patients curl up and assume a fetal position, tensing the abdominal muscles, inner thigh muscles, and pelvic floor muscles. Over time, these muscles come to maintain a short, tight, and painful state. (Hence the need for physical therapy to undo this decade-long pattern.)

Endometriosis implants on or near the gastrointestinal tract tug on fascia and muscles and commonly cause constipation, leading women to further overwork the pelvic floor muscles. In the case of diarrhea-predominant dysfunction, our patients squeeze pelvic floor muscles to prevent leakage. And in the case of urinary urgency, they squeeze muscles to release urine that isn’t really there.

As the chronic inflammation of the disease grows, and as pain worsens, the patient is increasingly in sympathetic overdrive (also known as ”fight or flight”), as opposed to a parasympathetic state (also known as “rest and digest”). The bowel’s motility slows, allowing the bacteria of the small intestine to grow beyond what is normal, leading to SIBO, a condition increasingly recognized by gastroenterologists and others that can impede nutrient absorption and cause bloat and pain and exacerbate constipation and diarrhea.

Key to my conceptualization of pain was a review published in 2011 by Pam Stratton, MD, of the National Institutes of Health, and Karen J. Berkley, PhD, then of Florida State University, on chronic pain and endometriosis.4 They detailed how endometriotic lesions can develop their own nerve supply that interacts directly and in a two-way fashion with the CNS – and how the lesions can engage the nervous system in ways that create comorbid conditions and pain that becomes “independent of the disease itself.”

Sensitized peripheral nerve fibers innervating a deeply infiltrating lesion on the left uterosacral ligament, for instance, can sensitize neurons in the spinal sacral segment. Branches of these nerve fibers can extend to other segments of the spinal cord, and, once sensitized themselves, turn on neurons in these other segments. There is a resultant remodeling of the central nervous system, in essence, and what is called “remote central sensitization.” The CNS becomes independent from peripheral neural processes.

I now explain to both patients and physicians that those who have had endometriosis for years have had an enduring “hand on the stove,” with a persistent signal to the CNS. Tight muscles are a hand on the stove, painful bladder syndrome is another hand on the stove, and SIBO is yet another. So are anxiety and depression.

The CNS becomes so upregulated and overloaded that messages branch out through the spinal cord to other available pathways and to other organs, muscles, and nerves. The CNS also starts firing on its own – and once it becomes its own pain generator, taking one hand off the stove (for instance, excising implants) while leaving multiple other hands on the hot stove won’t remove all pain. We must downregulate the CNS more broadly.

As I began addressing pain generators and instigators of CNS sensitization – and waiting for excision surgery until the CNS had sufficiently cooled – I saw that my patients had a better chance of more significant and lasting pain relief.
 

 

 

Pearls for a multimodal approach

My initial physical exam includes an assessment of the pelvic floor for overly tight musculature. An abdominal exam will usually reveal whether there is asymmetry of the abdominal wall muscles, which typically informs me of the likelihood of tightness and pulling on either side of the pelvic anatomy. On the internal exam, then, the pelvic floor muscles can be palpated and assessed. These findings will guide my referrals and my discussions with patients about the value of pelvic floor physical therapy. The cervix should be in the midline of the vagina – equidistant from the left and right vaginal fornices. If the cervix is pulled away from this midline, and a palpation of a thickened uterosacral ligament reproduces pain, endometriosis is 90% likely.

Patients who report significant “burning” pain that’s suggestive of neuropathic pain should be referred to a physical medicine rehabilitation physician or a pain specialist who can help downregulate their CNS. And patients who have symptoms of depression, anxiety disorders (including obsessive-compulsive disorder), or posttraumatic stress disorder should be referred to pain therapists, psychologists, or other mental health professionals, preferably well before surgery. I will also often discuss mindfulness practices and give my patients “meditation challenges” to achieve during the presurgical phase.

Additional points of emphasis about a multidisciplinary, multimodal approach include:

Advanced pelvic floor therapy: Therapists with specialized training in pelvic health and manual therapy utilize a range of techniques and modalities to release tension in affected muscles, fascia, nerves, and bone, and in doing so, they help to downregulate the CNS. Myofascial release, myofascial trigger point release, neural mobilization, and visceral mobilization are among these techniques. In addition to using manual therapy, many of these therapists may also employ neuromuscular reeducation and other techniques that will be helpful for the longer term.

It is important to identify physical therapists who have training in this approach; women with endometriosis often have a history of treatment by physical therapists whose focus is on incontinence and muscle strengthening (that is, Kegel exercises), which is the opposite of what endometriosis patients need.

Treating SIBO: Symptoms commonly associated with SIBO often overlap with symptoms of irritable bowel syndrome (IBS) – namely constipation, diarrhea (or both), and bloating. Indeed, many patients with undiagnosed endometriosis have been diagnosed with IBS. I send every patient who has one of these symptoms for SIBO breath testing, which utilizes carbohydrate substrates (glucose or lactulose) and measures hydrogen and/or methane in the breath.

SIBO is typically treated with rifampin, which stays in the small bowel and will not negatively affect beneficial bacteria, with or without neomycin. Gastroenterologists with more integrative practices also consider the use of herbals in addition to – or instead of – antibiotics. It can sometimes take months or a couple of years to correct SIBO, depending on how long the patient has been affected, but with presurgical diagnosis and a start on treatment, we can remove or at least tone down another instigator of CNS sensitization.

I estimate that 80% of my patients have tested positive for SIBO. Notably, in a testament to the systemic nature of endometriosis, a study published in 2009 of 355 women undergoing operative laparoscopy for suspected endometriosis found that 90% had gastrointestinal symptoms, but only 7.6% of the vast majority whose endometriosis was confirmed were found to have endometrial implants on the bowel itself.5

Addressing bladder issues: I routinely administer the PUF (Pain, Urgency, Frequency) questionnaire as part of my intake package and follow it up with conversation. For just about every patient with painful bladder syndrome, pelvic floor physical therapy in combination with a low-acid, low-potassium diet will work effectively together to reduce symptoms and pain. The IC Network offers a helpful food list, and patients can be counseled to choose foods that are also anti-inflammatory. When referrals to a urologist for bladder instillations are possible, these can be helpful as well.

Our communication with patients

Our patients need to have their symptoms and pain validated and to understand why we’re recommending these measures before surgery. Some education is necessary. Few patients will go to an integrative nutritionist, for example, if we just write a referral without explaining how years of inflammation and disruption in the gut can affect the whole body – including mental health – and that it can be corrected over time.

Also necessary is an appreciation of the fact that patients with delayed diagnoses have lived with gastrointestinal and other symptoms and patterns for so long – and often have mothers whose endometriosis caused similar symptoms – that some of their own experiences can seem almost “normal.” A patient whose mother had bowel movements every 7 days may think that 4-5 day intervals are acceptable, for instance. This means we have to carefully consider how we ask our questions.

I always ask my patients as we’re going into surgery, what percentage better are you? I’ve long aimed for at least 30% improvement, but most of the time, with pelvic floor therapy and as many other pain-generator–focused measures as possible, we’re getting them 70% better.

Excision surgery will remove the inflammation that has helped fuel the SIBO and other coconditions. Then, everything done to prepare the body must continue for some time. Certain practices, such as eating an anti-inflammatory diet, should be lifelong.

One day, it is hoped, a pediatrician or other physician will suspect endometriosis early on. The patient will see the surgeon within several months of the onset of pain, and we won’t need to unravel layers of pain generation and CNS upregulation before operating. But until this happens and we shorten the diagnostic delay, we must consider the benefits of presurgical preparation.
 

References

1. Orbuch I, Stein A. Beating Endo: How to Reclaim Your Life From Endometriosis. (New York: HarperCollins, 2019).

2. Healey M et al. J Minim Invasive Gynecol. 2014;21(6):999-1004.

3. Pundir J et al. J Minim Invasive Gynecol. 2017;24(5):747-56.

4. Stratton P, Berkley KJ. Hum Repro Update. 2011;17(3):327-46.

5. Maroun P et al. Aust N Z J Obstet Gynaecol. 2009;49(4):411-4.

Dr. Orbuch is a minimally invasive gynecologic surgeon in Los Angeles who specializes in endometriosis. She has no conflicts of interest to report.

Publications
Topics
Sections

Introduction: The preoperative evaluation for endometriosis – more than meets the eye

It is well known that it often takes 6-10 years for endometriosis to be diagnosed in patients who have the disease, depending on where the patient lives. I certainly am not surprised. During my residency at Parkland Memorial Hospital, if a patient had chronic pelvic pain and no fibroids, her diagnosis was usually pelvic inflammatory disease. Later, during my fellowship in reproductive endocrinology at the University of Pennsylvania, the diagnosis became endometriosis.

As I gained more interest and expertise in the treatment of endometriosis, I became aware of several articles concluding that if a woman sought treatment for chronic pelvic pain with an internist, the diagnosis would be irritable bowel syndrome (IBS); with a urologist, it would be interstitial cystitis; and with a gynecologist, endometriosis. Moreover, there is an increased propensity for IBS and IC in patients with endometriosis. There also is an increased risk of small intestine bacterial overgrowth (SIBO), as noted by our guest author for this latest installment of the Master Class in Gynecologic Surgery, Iris Orbuch, MD.

Like our guest author, I have also noted increased risk of pelvic floor myalgia. Dr. Orbuch clearly outlines why this occurs. In fact, we can now understand why many patients have multiple pelvic pain–inducing issues compounding their pain secondary to endometriosis and leading to remodeling of the central nervous system. Therefore, it certainly makes sense to follow Dr. Orbuch’s recommendation for a multidisciplinary pre- and postsurgical approach “to downregulate the pain generators.”

Dr. Orbuch is a minimally invasive gynecologic surgeon in Los Angeles who specializes in the treatment of patients diagnosed with endometriosis. Dr. Orbuch serves on the Board of Directors of the Foundation of the American Association of Gynecologic Laparoscopists and has served as the chair of the AAGL’s Special Interest Group on Endometriosis and Reproductive Surgery. She is the coauthor of the book “Beating Endo – How to Reclaim Your Life From Endometriosis” (New York: HarperCollins; 2019). The book is written for patients but addresses many issues discussed in this installment of the Master Class in Gynecologic Surgery.

Dr. Miller, MD, FACOG, is professor of obstetrics and gynecology, department of clinical sciences, Rosalind Franklin University of Medicine and Science, North Chicago. He has no conflicts of interest to report.

 

 

Patients with endometriosis and the all-too-often decade-long diagnostic delay have a variety of coexisting conditions that are pain generators – from painful bladder syndrome and pelvic floor dysfunction to a small intestine bacterial system that is significantly upregulated and sensitized.

For optimal surgical outcomes, and to help our patients recover from years of this inflammatory, systemic disease, we must treat our patients holistically and work to downregulate their pain as much as possible before excision surgery. I work with patients a few months prior to surgery, often for 4-5 months, during which time they not only see me for informative follow-ups, but also pelvic floor physical therapists, gastroenterologists, mental health professionals, integrative nutritionists, and physiatrists or pain specialists, depending on their needs.1

By identifying coexisting conditions in an initial consult and employing a presurgical multidisciplinary approach to downregulate the pain generators, my patients recover well from excision surgery, with greater and faster relief from pain, compared with those using standard approaches, and with little to no use of opioids.

At a minimum, given the unfortunate time constraints and productivity demands of working within health systems – and considering that surgeries are often scheduled a couple of months out – the surgeon could ensure that patients are engaged in at least 6-8 weeks of pelvic floor physical therapy before surgery to sufficiently lengthen the pelvic muscles and loosen surrounding fascia.

Short, tight pelvic floor muscles are almost universal in patients with delayed diagnosis of endometriosis and are significant generators of pain.
 

Appreciating sequelae of diagnostic delay

After my fellowship in advanced laparoscopic and pelvic surgery with Harry Reich, MD, and C. Y. Liu, MD, pioneers of endometriosis excision surgery, and as I did my residency in the early 2000s, I noticed puzzlement in the literature about why some patients still had lasting pain after thorough excision.

I didn’t doubt the efficacy of excision. It is the cornerstone of treatment, and at least one randomized double-blind trial2 and a systematic review and meta-analysis3 have demonstrated its superior efficacy over ablation in symptom reduction. What I did doubt was any presumption that surgery alone was enough. I knew there was more to healing when a disease process wreaks havoc on the body for more than a decade and that there were other generators of pain in addition to the endometriosis implants themselves.

As I began to focus on endometriosis in my own surgical practice, I strove to detect and treat endometriosis in teens. But in those patients with longstanding disease, I recognized patterns and began to more fully appreciate the systemic sequelae of endometriosis.

To cope with dysmenorrhea, patients curl up and assume a fetal position, tensing the abdominal muscles, inner thigh muscles, and pelvic floor muscles. Over time, these muscles come to maintain a short, tight, and painful state. (Hence the need for physical therapy to undo this decade-long pattern.)

Endometriosis implants on or near the gastrointestinal tract tug on fascia and muscles and commonly cause constipation, leading women to further overwork the pelvic floor muscles. In the case of diarrhea-predominant dysfunction, our patients squeeze pelvic floor muscles to prevent leakage. And in the case of urinary urgency, they squeeze muscles to release urine that isn’t really there.

As the chronic inflammation of the disease grows, and as pain worsens, the patient is increasingly in sympathetic overdrive (also known as ”fight or flight”), as opposed to a parasympathetic state (also known as “rest and digest”). The bowel’s motility slows, allowing the bacteria of the small intestine to grow beyond what is normal, leading to SIBO, a condition increasingly recognized by gastroenterologists and others that can impede nutrient absorption and cause bloat and pain and exacerbate constipation and diarrhea.

Key to my conceptualization of pain was a review published in 2011 by Pam Stratton, MD, of the National Institutes of Health, and Karen J. Berkley, PhD, then of Florida State University, on chronic pain and endometriosis.4 They detailed how endometriotic lesions can develop their own nerve supply that interacts directly and in a two-way fashion with the CNS – and how the lesions can engage the nervous system in ways that create comorbid conditions and pain that becomes “independent of the disease itself.”

Sensitized peripheral nerve fibers innervating a deeply infiltrating lesion on the left uterosacral ligament, for instance, can sensitize neurons in the spinal sacral segment. Branches of these nerve fibers can extend to other segments of the spinal cord, and, once sensitized themselves, turn on neurons in these other segments. There is a resultant remodeling of the central nervous system, in essence, and what is called “remote central sensitization.” The CNS becomes independent from peripheral neural processes.

I now explain to both patients and physicians that those who have had endometriosis for years have had an enduring “hand on the stove,” with a persistent signal to the CNS. Tight muscles are a hand on the stove, painful bladder syndrome is another hand on the stove, and SIBO is yet another. So are anxiety and depression.

The CNS becomes so upregulated and overloaded that messages branch out through the spinal cord to other available pathways and to other organs, muscles, and nerves. The CNS also starts firing on its own – and once it becomes its own pain generator, taking one hand off the stove (for instance, excising implants) while leaving multiple other hands on the hot stove won’t remove all pain. We must downregulate the CNS more broadly.

As I began addressing pain generators and instigators of CNS sensitization – and waiting for excision surgery until the CNS had sufficiently cooled – I saw that my patients had a better chance of more significant and lasting pain relief.
 

 

 

Pearls for a multimodal approach

My initial physical exam includes an assessment of the pelvic floor for overly tight musculature. An abdominal exam will usually reveal whether there is asymmetry of the abdominal wall muscles, which typically informs me of the likelihood of tightness and pulling on either side of the pelvic anatomy. On the internal exam, then, the pelvic floor muscles can be palpated and assessed. These findings will guide my referrals and my discussions with patients about the value of pelvic floor physical therapy. The cervix should be in the midline of the vagina – equidistant from the left and right vaginal fornices. If the cervix is pulled away from this midline, and a palpation of a thickened uterosacral ligament reproduces pain, endometriosis is 90% likely.

Patients who report significant “burning” pain that’s suggestive of neuropathic pain should be referred to a physical medicine rehabilitation physician or a pain specialist who can help downregulate their CNS. And patients who have symptoms of depression, anxiety disorders (including obsessive-compulsive disorder), or posttraumatic stress disorder should be referred to pain therapists, psychologists, or other mental health professionals, preferably well before surgery. I will also often discuss mindfulness practices and give my patients “meditation challenges” to achieve during the presurgical phase.

Additional points of emphasis about a multidisciplinary, multimodal approach include:

Advanced pelvic floor therapy: Therapists with specialized training in pelvic health and manual therapy utilize a range of techniques and modalities to release tension in affected muscles, fascia, nerves, and bone, and in doing so, they help to downregulate the CNS. Myofascial release, myofascial trigger point release, neural mobilization, and visceral mobilization are among these techniques. In addition to using manual therapy, many of these therapists may also employ neuromuscular reeducation and other techniques that will be helpful for the longer term.

It is important to identify physical therapists who have training in this approach; women with endometriosis often have a history of treatment by physical therapists whose focus is on incontinence and muscle strengthening (that is, Kegel exercises), which is the opposite of what endometriosis patients need.

Treating SIBO: Symptoms commonly associated with SIBO often overlap with symptoms of irritable bowel syndrome (IBS) – namely constipation, diarrhea (or both), and bloating. Indeed, many patients with undiagnosed endometriosis have been diagnosed with IBS. I send every patient who has one of these symptoms for SIBO breath testing, which utilizes carbohydrate substrates (glucose or lactulose) and measures hydrogen and/or methane in the breath.

SIBO is typically treated with rifampin, which stays in the small bowel and will not negatively affect beneficial bacteria, with or without neomycin. Gastroenterologists with more integrative practices also consider the use of herbals in addition to – or instead of – antibiotics. It can sometimes take months or a couple of years to correct SIBO, depending on how long the patient has been affected, but with presurgical diagnosis and a start on treatment, we can remove or at least tone down another instigator of CNS sensitization.

I estimate that 80% of my patients have tested positive for SIBO. Notably, in a testament to the systemic nature of endometriosis, a study published in 2009 of 355 women undergoing operative laparoscopy for suspected endometriosis found that 90% had gastrointestinal symptoms, but only 7.6% of the vast majority whose endometriosis was confirmed were found to have endometrial implants on the bowel itself.5

Addressing bladder issues: I routinely administer the PUF (Pain, Urgency, Frequency) questionnaire as part of my intake package and follow it up with conversation. For just about every patient with painful bladder syndrome, pelvic floor physical therapy in combination with a low-acid, low-potassium diet will work effectively together to reduce symptoms and pain. The IC Network offers a helpful food list, and patients can be counseled to choose foods that are also anti-inflammatory. When referrals to a urologist for bladder instillations are possible, these can be helpful as well.

Our communication with patients

Our patients need to have their symptoms and pain validated and to understand why we’re recommending these measures before surgery. Some education is necessary. Few patients will go to an integrative nutritionist, for example, if we just write a referral without explaining how years of inflammation and disruption in the gut can affect the whole body – including mental health – and that it can be corrected over time.

Also necessary is an appreciation of the fact that patients with delayed diagnoses have lived with gastrointestinal and other symptoms and patterns for so long – and often have mothers whose endometriosis caused similar symptoms – that some of their own experiences can seem almost “normal.” A patient whose mother had bowel movements every 7 days may think that 4-5 day intervals are acceptable, for instance. This means we have to carefully consider how we ask our questions.

I always ask my patients as we’re going into surgery, what percentage better are you? I’ve long aimed for at least 30% improvement, but most of the time, with pelvic floor therapy and as many other pain-generator–focused measures as possible, we’re getting them 70% better.

Excision surgery will remove the inflammation that has helped fuel the SIBO and other coconditions. Then, everything done to prepare the body must continue for some time. Certain practices, such as eating an anti-inflammatory diet, should be lifelong.

One day, it is hoped, a pediatrician or other physician will suspect endometriosis early on. The patient will see the surgeon within several months of the onset of pain, and we won’t need to unravel layers of pain generation and CNS upregulation before operating. But until this happens and we shorten the diagnostic delay, we must consider the benefits of presurgical preparation.
 

References

1. Orbuch I, Stein A. Beating Endo: How to Reclaim Your Life From Endometriosis. (New York: HarperCollins, 2019).

2. Healey M et al. J Minim Invasive Gynecol. 2014;21(6):999-1004.

3. Pundir J et al. J Minim Invasive Gynecol. 2017;24(5):747-56.

4. Stratton P, Berkley KJ. Hum Repro Update. 2011;17(3):327-46.

5. Maroun P et al. Aust N Z J Obstet Gynaecol. 2009;49(4):411-4.

Dr. Orbuch is a minimally invasive gynecologic surgeon in Los Angeles who specializes in endometriosis. She has no conflicts of interest to report.

Introduction: The preoperative evaluation for endometriosis – more than meets the eye

It is well known that it often takes 6-10 years for endometriosis to be diagnosed in patients who have the disease, depending on where the patient lives. I certainly am not surprised. During my residency at Parkland Memorial Hospital, if a patient had chronic pelvic pain and no fibroids, her diagnosis was usually pelvic inflammatory disease. Later, during my fellowship in reproductive endocrinology at the University of Pennsylvania, the diagnosis became endometriosis.

As I gained more interest and expertise in the treatment of endometriosis, I became aware of several articles concluding that if a woman sought treatment for chronic pelvic pain with an internist, the diagnosis would be irritable bowel syndrome (IBS); with a urologist, it would be interstitial cystitis; and with a gynecologist, endometriosis. Moreover, there is an increased propensity for IBS and IC in patients with endometriosis. There also is an increased risk of small intestine bacterial overgrowth (SIBO), as noted by our guest author for this latest installment of the Master Class in Gynecologic Surgery, Iris Orbuch, MD.

Like our guest author, I have also noted increased risk of pelvic floor myalgia. Dr. Orbuch clearly outlines why this occurs. In fact, we can now understand why many patients have multiple pelvic pain–inducing issues compounding their pain secondary to endometriosis and leading to remodeling of the central nervous system. Therefore, it certainly makes sense to follow Dr. Orbuch’s recommendation for a multidisciplinary pre- and postsurgical approach “to downregulate the pain generators.”

Dr. Orbuch is a minimally invasive gynecologic surgeon in Los Angeles who specializes in the treatment of patients diagnosed with endometriosis. Dr. Orbuch serves on the Board of Directors of the Foundation of the American Association of Gynecologic Laparoscopists and has served as the chair of the AAGL’s Special Interest Group on Endometriosis and Reproductive Surgery. She is the coauthor of the book “Beating Endo – How to Reclaim Your Life From Endometriosis” (New York: HarperCollins; 2019). The book is written for patients but addresses many issues discussed in this installment of the Master Class in Gynecologic Surgery.

Dr. Miller, MD, FACOG, is professor of obstetrics and gynecology, department of clinical sciences, Rosalind Franklin University of Medicine and Science, North Chicago. He has no conflicts of interest to report.

 

 

Patients with endometriosis and the all-too-often decade-long diagnostic delay have a variety of coexisting conditions that are pain generators – from painful bladder syndrome and pelvic floor dysfunction to a small intestine bacterial system that is significantly upregulated and sensitized.

For optimal surgical outcomes, and to help our patients recover from years of this inflammatory, systemic disease, we must treat our patients holistically and work to downregulate their pain as much as possible before excision surgery. I work with patients a few months prior to surgery, often for 4-5 months, during which time they not only see me for informative follow-ups, but also pelvic floor physical therapists, gastroenterologists, mental health professionals, integrative nutritionists, and physiatrists or pain specialists, depending on their needs.1

By identifying coexisting conditions in an initial consult and employing a presurgical multidisciplinary approach to downregulate the pain generators, my patients recover well from excision surgery, with greater and faster relief from pain, compared with those using standard approaches, and with little to no use of opioids.

At a minimum, given the unfortunate time constraints and productivity demands of working within health systems – and considering that surgeries are often scheduled a couple of months out – the surgeon could ensure that patients are engaged in at least 6-8 weeks of pelvic floor physical therapy before surgery to sufficiently lengthen the pelvic muscles and loosen surrounding fascia.

Short, tight pelvic floor muscles are almost universal in patients with delayed diagnosis of endometriosis and are significant generators of pain.
 

Appreciating sequelae of diagnostic delay

After my fellowship in advanced laparoscopic and pelvic surgery with Harry Reich, MD, and C. Y. Liu, MD, pioneers of endometriosis excision surgery, and as I did my residency in the early 2000s, I noticed puzzlement in the literature about why some patients still had lasting pain after thorough excision.

I didn’t doubt the efficacy of excision. It is the cornerstone of treatment, and at least one randomized double-blind trial2 and a systematic review and meta-analysis3 have demonstrated its superior efficacy over ablation in symptom reduction. What I did doubt was any presumption that surgery alone was enough. I knew there was more to healing when a disease process wreaks havoc on the body for more than a decade and that there were other generators of pain in addition to the endometriosis implants themselves.

As I began to focus on endometriosis in my own surgical practice, I strove to detect and treat endometriosis in teens. But in those patients with longstanding disease, I recognized patterns and began to more fully appreciate the systemic sequelae of endometriosis.

To cope with dysmenorrhea, patients curl up and assume a fetal position, tensing the abdominal muscles, inner thigh muscles, and pelvic floor muscles. Over time, these muscles come to maintain a short, tight, and painful state. (Hence the need for physical therapy to undo this decade-long pattern.)

Endometriosis implants on or near the gastrointestinal tract tug on fascia and muscles and commonly cause constipation, leading women to further overwork the pelvic floor muscles. In the case of diarrhea-predominant dysfunction, our patients squeeze pelvic floor muscles to prevent leakage. And in the case of urinary urgency, they squeeze muscles to release urine that isn’t really there.

As the chronic inflammation of the disease grows, and as pain worsens, the patient is increasingly in sympathetic overdrive (also known as ”fight or flight”), as opposed to a parasympathetic state (also known as “rest and digest”). The bowel’s motility slows, allowing the bacteria of the small intestine to grow beyond what is normal, leading to SIBO, a condition increasingly recognized by gastroenterologists and others that can impede nutrient absorption and cause bloat and pain and exacerbate constipation and diarrhea.

Key to my conceptualization of pain was a review published in 2011 by Pam Stratton, MD, of the National Institutes of Health, and Karen J. Berkley, PhD, then of Florida State University, on chronic pain and endometriosis.4 They detailed how endometriotic lesions can develop their own nerve supply that interacts directly and in a two-way fashion with the CNS – and how the lesions can engage the nervous system in ways that create comorbid conditions and pain that becomes “independent of the disease itself.”

Sensitized peripheral nerve fibers innervating a deeply infiltrating lesion on the left uterosacral ligament, for instance, can sensitize neurons in the spinal sacral segment. Branches of these nerve fibers can extend to other segments of the spinal cord, and, once sensitized themselves, turn on neurons in these other segments. There is a resultant remodeling of the central nervous system, in essence, and what is called “remote central sensitization.” The CNS becomes independent from peripheral neural processes.

I now explain to both patients and physicians that those who have had endometriosis for years have had an enduring “hand on the stove,” with a persistent signal to the CNS. Tight muscles are a hand on the stove, painful bladder syndrome is another hand on the stove, and SIBO is yet another. So are anxiety and depression.

The CNS becomes so upregulated and overloaded that messages branch out through the spinal cord to other available pathways and to other organs, muscles, and nerves. The CNS also starts firing on its own – and once it becomes its own pain generator, taking one hand off the stove (for instance, excising implants) while leaving multiple other hands on the hot stove won’t remove all pain. We must downregulate the CNS more broadly.

As I began addressing pain generators and instigators of CNS sensitization – and waiting for excision surgery until the CNS had sufficiently cooled – I saw that my patients had a better chance of more significant and lasting pain relief.
 

 

 

Pearls for a multimodal approach

My initial physical exam includes an assessment of the pelvic floor for overly tight musculature. An abdominal exam will usually reveal whether there is asymmetry of the abdominal wall muscles, which typically informs me of the likelihood of tightness and pulling on either side of the pelvic anatomy. On the internal exam, then, the pelvic floor muscles can be palpated and assessed. These findings will guide my referrals and my discussions with patients about the value of pelvic floor physical therapy. The cervix should be in the midline of the vagina – equidistant from the left and right vaginal fornices. If the cervix is pulled away from this midline, and a palpation of a thickened uterosacral ligament reproduces pain, endometriosis is 90% likely.

Patients who report significant “burning” pain that’s suggestive of neuropathic pain should be referred to a physical medicine rehabilitation physician or a pain specialist who can help downregulate their CNS. And patients who have symptoms of depression, anxiety disorders (including obsessive-compulsive disorder), or posttraumatic stress disorder should be referred to pain therapists, psychologists, or other mental health professionals, preferably well before surgery. I will also often discuss mindfulness practices and give my patients “meditation challenges” to achieve during the presurgical phase.

Additional points of emphasis about a multidisciplinary, multimodal approach include:

Advanced pelvic floor therapy: Therapists with specialized training in pelvic health and manual therapy utilize a range of techniques and modalities to release tension in affected muscles, fascia, nerves, and bone, and in doing so, they help to downregulate the CNS. Myofascial release, myofascial trigger point release, neural mobilization, and visceral mobilization are among these techniques. In addition to using manual therapy, many of these therapists may also employ neuromuscular reeducation and other techniques that will be helpful for the longer term.

It is important to identify physical therapists who have training in this approach; women with endometriosis often have a history of treatment by physical therapists whose focus is on incontinence and muscle strengthening (that is, Kegel exercises), which is the opposite of what endometriosis patients need.

Treating SIBO: Symptoms commonly associated with SIBO often overlap with symptoms of irritable bowel syndrome (IBS) – namely constipation, diarrhea (or both), and bloating. Indeed, many patients with undiagnosed endometriosis have been diagnosed with IBS. I send every patient who has one of these symptoms for SIBO breath testing, which utilizes carbohydrate substrates (glucose or lactulose) and measures hydrogen and/or methane in the breath.

SIBO is typically treated with rifampin, which stays in the small bowel and will not negatively affect beneficial bacteria, with or without neomycin. Gastroenterologists with more integrative practices also consider the use of herbals in addition to – or instead of – antibiotics. It can sometimes take months or a couple of years to correct SIBO, depending on how long the patient has been affected, but with presurgical diagnosis and a start on treatment, we can remove or at least tone down another instigator of CNS sensitization.

I estimate that 80% of my patients have tested positive for SIBO. Notably, in a testament to the systemic nature of endometriosis, a study published in 2009 of 355 women undergoing operative laparoscopy for suspected endometriosis found that 90% had gastrointestinal symptoms, but only 7.6% of the vast majority whose endometriosis was confirmed were found to have endometrial implants on the bowel itself.5

Addressing bladder issues: I routinely administer the PUF (Pain, Urgency, Frequency) questionnaire as part of my intake package and follow it up with conversation. For just about every patient with painful bladder syndrome, pelvic floor physical therapy in combination with a low-acid, low-potassium diet will work effectively together to reduce symptoms and pain. The IC Network offers a helpful food list, and patients can be counseled to choose foods that are also anti-inflammatory. When referrals to a urologist for bladder instillations are possible, these can be helpful as well.

Our communication with patients

Our patients need to have their symptoms and pain validated and to understand why we’re recommending these measures before surgery. Some education is necessary. Few patients will go to an integrative nutritionist, for example, if we just write a referral without explaining how years of inflammation and disruption in the gut can affect the whole body – including mental health – and that it can be corrected over time.

Also necessary is an appreciation of the fact that patients with delayed diagnoses have lived with gastrointestinal and other symptoms and patterns for so long – and often have mothers whose endometriosis caused similar symptoms – that some of their own experiences can seem almost “normal.” A patient whose mother had bowel movements every 7 days may think that 4-5 day intervals are acceptable, for instance. This means we have to carefully consider how we ask our questions.

I always ask my patients as we’re going into surgery, what percentage better are you? I’ve long aimed for at least 30% improvement, but most of the time, with pelvic floor therapy and as many other pain-generator–focused measures as possible, we’re getting them 70% better.

Excision surgery will remove the inflammation that has helped fuel the SIBO and other coconditions. Then, everything done to prepare the body must continue for some time. Certain practices, such as eating an anti-inflammatory diet, should be lifelong.

One day, it is hoped, a pediatrician or other physician will suspect endometriosis early on. The patient will see the surgeon within several months of the onset of pain, and we won’t need to unravel layers of pain generation and CNS upregulation before operating. But until this happens and we shorten the diagnostic delay, we must consider the benefits of presurgical preparation.
 

References

1. Orbuch I, Stein A. Beating Endo: How to Reclaim Your Life From Endometriosis. (New York: HarperCollins, 2019).

2. Healey M et al. J Minim Invasive Gynecol. 2014;21(6):999-1004.

3. Pundir J et al. J Minim Invasive Gynecol. 2017;24(5):747-56.

4. Stratton P, Berkley KJ. Hum Repro Update. 2011;17(3):327-46.

5. Maroun P et al. Aust N Z J Obstet Gynaecol. 2009;49(4):411-4.

Dr. Orbuch is a minimally invasive gynecologic surgeon in Los Angeles who specializes in endometriosis. She has no conflicts of interest to report.

Publications
Publications
Topics
Article Type
Sections
Disallow All Ads
Content Gating
No Gating (article Unlocked/Free)
Alternative CME
Disqus Comments
Default
Use ProPublica
Hide sidebar & use full width
render the right sidebar.
Conference Recap Checkbox
Not Conference Recap
Clinical Edge
Display the Slideshow in this Article
Medscape Article
Display survey writer
Reuters content
Disable Inline Native ads
WebMD Article

Red wine’s potential benefits for cardiovascular health

Article Type
Changed
Tue, 02/28/2023 - 13:41

In recent weeks, you may have noticed some familiar headlines about red wine and cardiovascular health. Why the sudden return of these stories? Because of an article recently published in the American Journal of Clinical Nutrition.

Funded in part by a grant from the São Paulo Research Foundation (FAPESP), the “Wine Flora Study” was carried out by prominent researchers from institutions in South America, Europe, and the United States: University of São Paulo; State University of Campinas, São Paulo; University of Brasília; University of Verona (Italy); Austrian Institute of Technology, Tulln; and Harvard Medical School, Boston. The team looked into the effects of red wine on gut flora and plasma levels of trimethylamine-N-oxide (TMAO). And what they found was quite interesting.
 

The study

Previous results have pointed to the beneficial effect that red wine has on the gut microbiome.

The Wine Flora Study involved 42 men (average age, 60 years) with documented coronary artery disease. The trial encompassed two 3-week interventions. In one, the participants consumed 250 mL of red wine per day; the red wine sample had an alcohol content (% v) of 12.75. The Brazilian Wine Institute produced and supplied the red wine: a 2014 Merlot bottled in August 2016 and customized for the study. The second intervention involved alcohol abstention.

Each intervention was preceded by a 2-week washout period. Because certain foods and drinks could interfere with the results, the participants were instructed not to consume alcoholic beverages, fermented foods (yogurt, kombucha, soy lecithin, kefir, sauerkraut, and other fermented vegetables), synthetic prebiotics (insulin, fructooligosaccharides), fiber, dairy, food polyphenols (grapes, grape juice, cranberries, strawberries), and probiotics.

At each intervention, the gut microbiota was analyzed via 16S ribosomal RNA highthroughput sequencing. This method makes it possible to identify bacterial species. The plasma metabolome of 20 randomly selected participants was evaluated by ultra–high-performance liquid chromatography with tandem mass spectrometry. In this method, liquid chromatography separates the compounds, and a mass spectrometer is used to analyze them.

One of the metabolites of interest was TMAO, which is produced from the trimethylamine released when gut bacteria process protein-rich foods. TMAO has been identified as playing a role in the development of atherosclerosis.
 

Results

After red wine consumption, there was significant remodeling of the gut microbiota, with a difference in beta diversity and predominance of Parasutterella, Ruminococcaceae, several Bacteroides species, and Prevotella.

Plasma metabolomic analysis revealed significant changes in metabolites after red wine consumption, consistent with improved redox homeostasis, which is involved in the oxidative stress that promotes atherosclerosis.

Plasma TMAO, however, did not differ between red wine intervention and alcohol abstention.
 

Implications

The researchers concluded that modulation of the gut microbiota may contribute to the putative cardiovascular benefits of moderate red wine consumption. But, as they were careful to point out in the very title of the study, a red wine intervention does not modify plasma TMAO. They also mentioned that the 3-week period may have been too short for the findings to serve as the basis for promoting any meaningful modification. In addition, the team emphasized that these data remain hypothesisgenerating and pave the way for future research.

In an interview with FAPESP, the study’s corresponding author, Protásio Lemos da Luz, MD, PhD, warned about the risks associated with drinking too much alcohol (> 8.5 oz., or 250 mL, of wine daily).

It should be kept in mind that, in Brazil, people do not drink nearly as much wine as they do beer or liquor. Furthermore, the evidence that is available does not provide confirmation of the existence or the extent of the protective health effects associated with light or moderate alcohol intake.

This article was translated from the Medscape Portuguese edition. A version appeared on Medscape.com.

Publications
Topics
Sections

In recent weeks, you may have noticed some familiar headlines about red wine and cardiovascular health. Why the sudden return of these stories? Because of an article recently published in the American Journal of Clinical Nutrition.

Funded in part by a grant from the São Paulo Research Foundation (FAPESP), the “Wine Flora Study” was carried out by prominent researchers from institutions in South America, Europe, and the United States: University of São Paulo; State University of Campinas, São Paulo; University of Brasília; University of Verona (Italy); Austrian Institute of Technology, Tulln; and Harvard Medical School, Boston. The team looked into the effects of red wine on gut flora and plasma levels of trimethylamine-N-oxide (TMAO). And what they found was quite interesting.
 

The study

Previous results have pointed to the beneficial effect that red wine has on the gut microbiome.

The Wine Flora Study involved 42 men (average age, 60 years) with documented coronary artery disease. The trial encompassed two 3-week interventions. In one, the participants consumed 250 mL of red wine per day; the red wine sample had an alcohol content (% v) of 12.75. The Brazilian Wine Institute produced and supplied the red wine: a 2014 Merlot bottled in August 2016 and customized for the study. The second intervention involved alcohol abstention.

Each intervention was preceded by a 2-week washout period. Because certain foods and drinks could interfere with the results, the participants were instructed not to consume alcoholic beverages, fermented foods (yogurt, kombucha, soy lecithin, kefir, sauerkraut, and other fermented vegetables), synthetic prebiotics (insulin, fructooligosaccharides), fiber, dairy, food polyphenols (grapes, grape juice, cranberries, strawberries), and probiotics.

At each intervention, the gut microbiota was analyzed via 16S ribosomal RNA highthroughput sequencing. This method makes it possible to identify bacterial species. The plasma metabolome of 20 randomly selected participants was evaluated by ultra–high-performance liquid chromatography with tandem mass spectrometry. In this method, liquid chromatography separates the compounds, and a mass spectrometer is used to analyze them.

One of the metabolites of interest was TMAO, which is produced from the trimethylamine released when gut bacteria process protein-rich foods. TMAO has been identified as playing a role in the development of atherosclerosis.
 

Results

After red wine consumption, there was significant remodeling of the gut microbiota, with a difference in beta diversity and predominance of Parasutterella, Ruminococcaceae, several Bacteroides species, and Prevotella.

Plasma metabolomic analysis revealed significant changes in metabolites after red wine consumption, consistent with improved redox homeostasis, which is involved in the oxidative stress that promotes atherosclerosis.

Plasma TMAO, however, did not differ between red wine intervention and alcohol abstention.
 

Implications

The researchers concluded that modulation of the gut microbiota may contribute to the putative cardiovascular benefits of moderate red wine consumption. But, as they were careful to point out in the very title of the study, a red wine intervention does not modify plasma TMAO. They also mentioned that the 3-week period may have been too short for the findings to serve as the basis for promoting any meaningful modification. In addition, the team emphasized that these data remain hypothesisgenerating and pave the way for future research.

In an interview with FAPESP, the study’s corresponding author, Protásio Lemos da Luz, MD, PhD, warned about the risks associated with drinking too much alcohol (> 8.5 oz., or 250 mL, of wine daily).

It should be kept in mind that, in Brazil, people do not drink nearly as much wine as they do beer or liquor. Furthermore, the evidence that is available does not provide confirmation of the existence or the extent of the protective health effects associated with light or moderate alcohol intake.

This article was translated from the Medscape Portuguese edition. A version appeared on Medscape.com.

In recent weeks, you may have noticed some familiar headlines about red wine and cardiovascular health. Why the sudden return of these stories? Because of an article recently published in the American Journal of Clinical Nutrition.

Funded in part by a grant from the São Paulo Research Foundation (FAPESP), the “Wine Flora Study” was carried out by prominent researchers from institutions in South America, Europe, and the United States: University of São Paulo; State University of Campinas, São Paulo; University of Brasília; University of Verona (Italy); Austrian Institute of Technology, Tulln; and Harvard Medical School, Boston. The team looked into the effects of red wine on gut flora and plasma levels of trimethylamine-N-oxide (TMAO). And what they found was quite interesting.
 

The study

Previous results have pointed to the beneficial effect that red wine has on the gut microbiome.

The Wine Flora Study involved 42 men (average age, 60 years) with documented coronary artery disease. The trial encompassed two 3-week interventions. In one, the participants consumed 250 mL of red wine per day; the red wine sample had an alcohol content (% v) of 12.75. The Brazilian Wine Institute produced and supplied the red wine: a 2014 Merlot bottled in August 2016 and customized for the study. The second intervention involved alcohol abstention.

Each intervention was preceded by a 2-week washout period. Because certain foods and drinks could interfere with the results, the participants were instructed not to consume alcoholic beverages, fermented foods (yogurt, kombucha, soy lecithin, kefir, sauerkraut, and other fermented vegetables), synthetic prebiotics (insulin, fructooligosaccharides), fiber, dairy, food polyphenols (grapes, grape juice, cranberries, strawberries), and probiotics.

At each intervention, the gut microbiota was analyzed via 16S ribosomal RNA highthroughput sequencing. This method makes it possible to identify bacterial species. The plasma metabolome of 20 randomly selected participants was evaluated by ultra–high-performance liquid chromatography with tandem mass spectrometry. In this method, liquid chromatography separates the compounds, and a mass spectrometer is used to analyze them.

One of the metabolites of interest was TMAO, which is produced from the trimethylamine released when gut bacteria process protein-rich foods. TMAO has been identified as playing a role in the development of atherosclerosis.
 

Results

After red wine consumption, there was significant remodeling of the gut microbiota, with a difference in beta diversity and predominance of Parasutterella, Ruminococcaceae, several Bacteroides species, and Prevotella.

Plasma metabolomic analysis revealed significant changes in metabolites after red wine consumption, consistent with improved redox homeostasis, which is involved in the oxidative stress that promotes atherosclerosis.

Plasma TMAO, however, did not differ between red wine intervention and alcohol abstention.
 

Implications

The researchers concluded that modulation of the gut microbiota may contribute to the putative cardiovascular benefits of moderate red wine consumption. But, as they were careful to point out in the very title of the study, a red wine intervention does not modify plasma TMAO. They also mentioned that the 3-week period may have been too short for the findings to serve as the basis for promoting any meaningful modification. In addition, the team emphasized that these data remain hypothesisgenerating and pave the way for future research.

In an interview with FAPESP, the study’s corresponding author, Protásio Lemos da Luz, MD, PhD, warned about the risks associated with drinking too much alcohol (> 8.5 oz., or 250 mL, of wine daily).

It should be kept in mind that, in Brazil, people do not drink nearly as much wine as they do beer or liquor. Furthermore, the evidence that is available does not provide confirmation of the existence or the extent of the protective health effects associated with light or moderate alcohol intake.

This article was translated from the Medscape Portuguese edition. A version appeared on Medscape.com.

Publications
Publications
Topics
Article Type
Sections
Disallow All Ads
Content Gating
No Gating (article Unlocked/Free)
Alternative CME
Disqus Comments
Default
Use ProPublica
Hide sidebar & use full width
render the right sidebar.
Conference Recap Checkbox
Not Conference Recap
Clinical Edge
Display the Slideshow in this Article
Medscape Article
Display survey writer
Reuters content
Disable Inline Native ads
WebMD Article